Sei sulla pagina 1di 87

Learning outcomes

By the end of this tutorial, you will be able to:


 demonstrate knowledge of AUB terminology and the FIGO classification system
(PALM-COEIN) for causes of AUB and understand the risks and benefits of
treatment options
 identify ‘red flag’ features requiring secondary care referral
 describe the principles of uterine and endometrial assessment using pelvic
ultrasound, endometrial biopsy and hysteroscopy and identify the indications for
using other imaging modalities in selected groups
 interpret the hierarchical three-step treatment approach for HMB
 recognize the risks and benefits of: operative hysteroscopic procedures,
endometrial ablation, uterine artery embolization and MRI-guided focused
ultrasound ablation
 counsel women on treatment options explaining the nature, risks and benefits of
medical and surgical treatments
 recognize the need for appropriate referral for more complex cases for detailed
evaluation
 use appropriate referral pathways and local protocols if AUB or abnormal findings
are identified.

Menstruation

Menstruation is a
woman's
monthly bleeding
from her
reproductive
tract induced by hormonal changes of the menstrual cycle. The length of a menstrual
cycle is the time from the start of a period to the start of the next.

Beliefs derived from personal experience and cultural, social and educational influences
determine whether she perceives the menstrual blood loss to be 'normal' for her.

1
However, a 'normal' quantity of monthly blood loss (MBL) can be defined objectively for
the whole population.

Due to difficulties in determining when a menstrual period begins (e.g. spotting,


brown/pink discharge, continuous prolonged bleed), it is often difficult to differentiate
between a menstrual period and an intermenstrual bleed (IMB). In the main, the
aetiopathology and treatment of IMB differs from heavy menstrual bleeding (HMB).

Normal menstrual cycle

The menstrual cycle (as well as AUB) should be described according to four specific
symptomatic components (cycle frequency, duration, volume, regularity of cycle).

Frequency of menses (or length of menstrual cycle)

 Mean is 28 days (95% CI 24–38 days); frequent <24 days, infrequent >38 days.

As age increases, the menstrual cycle tends to shorten; age 13–19 years, mean cycle
length 35 days (90th centile range: 28–44 days), age 35–52 years, mean cycle length
28 days (90th centile range: 25–32 days).

As age increases, the frequency of irregular periods reduces. The frequency of irregular
periods is around 21% between the ages 15 and 19 years and reduces 11% between
ages 40 and 44 years.

Duration of menstruation

 Normal 4.5–8.0 days; prolonged >8 days, shortened <4.5 days.


 Mean is 5 days (95% CI 4.5–8 days).

As age increases, the duration of menstruation decreases. One study has reported that
the mean duration of menstruation changed from 3.9 days at aged 20, to 2.8 days at
40 years. Another study showed the average duration of menstruation was 4.7 days in
women aged between 13 to 17 years old and 4.1 days in women aged over 40 years.

Volume of monthly menstrual blood loss (ml)

 Mean 40 ml (95% CI 5–80 ml); heavy is >80 ml; light is <5 ml.

In women with no pre-existing menstrual problems, the normal amount of monthly


menstrual blood loss (MBL) is 25–50 ml. An early study showed that haemoglobin and

2
ferritin levels adversely changed at MBL levels beyond 80 ml and suggested this should
be considered the upper limit of the population average MBL.

A subsequent study showed that anaemia and iron depletion occurred at two points,
first around 60 ml MBL and then around 120 ml MBL. The study concluded that a
definition around 120 ml may be more useful for the management of women with HMB
as this correlated to when anaemia was most likely to occur.

Regularity of menstrual cycle (cycle to cycle variation over 12 months,


measured in days)

Regular cycle-to-cycle variation is between 2–20 days, irregular variation is >20 days or
absent.

Components used to describe type of AUB Normal limits (5th–95th


centile)
Frequency of menses (days) Frequent <24
Normal 24–38
Infrequent >38
Duration of flow (days) Prolonged >8.0
Normal 4.5–8.0
Shortened <4.5
Volume of monthly blood loss (ml) Heavy >80
Normal 5–80
Median 40
Light <5
Regularity of menses (cycle to cycle variation over 12 Regular variation: ± 2 to 20
months; in days) days
Irregular variation: >20 days
or absent

Connolly A, Jones SE. Nonmenstrual bleeding in woman under 40 years of age. The Obstetrician &
Gynaecologist 2004;6:153–8.

Warrilow G, Kirkham C, Ismail K, Wyatt K, Dimmock P, O’Brien S.Quantification of menstrual blood loss. The
Obstetrician & Gynaecologist 2004;6:88–92.

Fraser IS, Critchley HOD, Munro MG, Broder M. Can we achieve international agreement on terminologies
and definitions used to describe abnormalities of menstrual bleeding? Hum Reprod 2007;22:635–643.

Key points

Menstruation is controlled by the cyclical hormonal change in the menstrual


cycle.

Normal frequency is between 24–38 days.

3
Normal duration is 4.5–8 days.

Normal volume is between 5–80 ml.

Regular cycle-to-cycle variation is between 2–20 days.

Definition of HMB

Heavy menstrual bleeding (HMB) is clinically defined as menstrual blood loss (MBL)
that is subjectively considered to be excessive by the woman and interferes
with her physical, emotional, social and material quality of life.

Quantifying monthly menstrual blood loss (MBL) does not improve clinical care and is
not undertaken in modern clinical practice. MBL may be estimated directly (e.g. by
collecting all sanitary protection and eluting and quantifying blood by laboratory
techniques such as the alkaline haematin test) or indirectly (e.g. Pictorial Blood Loss
Assessment Chart [PBAC]; subjective measures).

Objective assessment: a PBAC score greater than or equal to 100 equates to a


sensitivity of 86%–91% and a specificity of 82–89% in predicting MBL greater than 80
ml by alkaline haematin test (gold standard reference technique).

Subjective assessment: subjective assessment of MBL combines information of


sanitary protection usage, flooding, clots, duration of menstruation and the woman's
personal opinion of her menstrual loss. Although this tends to be inaccurate, it is easy
to undertake in clinical practice and is the preferred method of assessing HMB.

Prevalence of HMB

HMB has a major adverse effect on the quality of life of many women. Overall, 3% of
premenopausal women experience HMB. However, this absolute risk is almost doubled
in woman aged 40–51. HMB accounts for around 15% of all secondary care
gynaecological referrals in the UK.

It is estimated that HMB costs the UK NHS around £100 million a year.

Greater awareness of HMB led by better


health education and expanding internet
resources has empowered women to seek
earlier medical advice and the expectation
of greater treatment choice.

Annual rate of women with HMB presenting to


service

4
Figure adapted from: National Institute of Health and Clinical Excellence. Heavy menstrual bleeding:
costing report. Implementing NICE guidance in England. London: NICE; 2007

Causes of HMB

Between 40–60% of women with HMB have no


uterine, endocrine, haematological or infective
pathology on investigation. These women were
formerly termed to have dysfunctional uterine
bleeding (DUB) of ovulatory (regular cycle) or
anovulatory (irregular cycle) type.

Most HMB is due to a combination of


coagulopathy, ovulatory or endometrial
dysfunction that does not require secondary
referral and treatment can be commenced in primary care. The term DUB should be
discarded and PALM-COEIN classification adopted.

Pathological causes of HMB include uterine fibroids (20–30%), uterine polyps (5–10%),
adenomyosis (5%); endometriosis rarely presents as AUB, but is identified in <5% of
cases of AUB.

Gynaecological malignancy rarely presents as HMB, but can present as prolonged


intermenstrual bleeding (IMB), postcoital bleeding (PCB), postmenopausal bleeding
(PMB) and as a pelvic mass.

Key points

Heavy menstrual bleeding is a subjective diagnosis as it is defined by the woman


based on how it interferes with her quality of life.

HMB affects 3% of premenopausal women.

Pathological causes of HMB include uterine fibroids (20–30%), uterine polyps (5–
10%), Adenomyosis (5%); endometriosis rarely presents as AUB, but is identified
in <5% of cases of AUB.

DUB should not be used to describe HMB.

Terminology Used to Describe AUB

Terms to be kept in new terminology

Abnormal Any menstrual bleeding from the uterus that is either abnormal in
uterine bleeding volume (excessive duration or heavy), regularity, timing (delayed

5
(AUB) or frequent) or is non-menstrual (IMB, PCB or PMB)

Heavy For clinical purposes, HMB is defined as excessive menstrual blood


menstrual loss leading to interference with the physical, emotional, social and
bleeding (HMB) material quality of life of a woman, and which can occur alone or in
combination with other symptoms. Adverse outcome is greater
in women with total MBL that exceeds 80 ml or menses
duration >7 days

Intermenstrual Uterine bleeding that occurs between clearly defined cyclic and
bleeding (IMB) predictable menses. Such bleeding may occur at random times or
may manifest in a predictable fashion at the same day in each
cycle

Postmenopausal Genital tract bleeding that recurs in a menopausal woman at least


bleeding (PMB) one year after cessation of cycles

Postcoital Non-menstrual genital tract bleeding immediately (or shortly after)


bleeding (PCB) intercourse

Chronic AUB AUB has been present for the majority of the past 6 months. In
most cases, chronic AUB is unlikely to require urgent immediate
clinical intervention

Acute AUB Excessive AUB bleeding that requires immediate intervention to


prevent further blood loss. Acute AUB may present in the context of
existing chronic AUB or might occur without such a history

Terms that should be discarded in new terminology

Menorrhagia Heavy menstrual bleeding that occurs at expected intervals of the


menstrual cycle (cycle length varying from 21 to 35 days)

Oligomenorrhoe Bleeding that occurs at intervals of >35 days and <6 months,
a usually caused by a prolonged follicular phase

Polymenorrhoea Regular bleeding at intervals of less than 3 weeks, which may be


caused by a luteal phase defect

Amenorrhoea No uterine bleeding for at least 6 months

Menometrorrha HMB at the usual time of menstrual periods and at other irregular
gia intervals

Metrorrhagia Uterine bleeding at irregular intervals, particularly between the


expected menstrual periods

6
Dysfunctional This may be ovulatory or anovulatory HMB. This is diagnosed after
uterine bleeding the exclusion of pregnancy, medications, iatrogenic causes, genital
(DUB) tract pathology and systemic conditions

 The terms listed in the table above, until now, have been widely accepted as
appropriate terms to describe AUB. However, there has been concern that such
terminology is poorly understood by both doctors and patients and is liable to
misinterpretation.

 An expert consensus panel on menstrual disorders (Fraser et al 2007) has


suggested that:

 terms such as menorrhagia, menometrorrhagia, metrorrhagia and dysfunctional


uterine bleeding be discarded
 AUB should be described according to four specific symptomatic components
(cycle frequency, duration, volume and regularity of cycle). For example, instead
of stating this woman has menorrhagia, one would state she has normal
frequency, prolonged duration, heavy menstrual bleeding (HMB) without any
variation between cycles. The terminology suggested by Fraser et al advises
using simple universally accepted terminology to describe the four cycle-
components:
o regularity should be specified as irregular, regular or absent
o frequency should be specified as frequent, normal or infrequent
o duration should be specified as prolonged, normal or shortened
o volume should be specified as heavy, normal or light.
In describing a cycle as above one would accept the ‘normal’ parameters
as outlined by Fraser et al (see normal menstrual cycle section).
 No specific guidance has been issued for the terms oligomenorrhoea,
polymenorrhoea and amenorrhoea. However, these are better described using
cycle frequency and regularity of cycle components.
Fraser IS, Critchley HOD, Munro MG, Broder M. Can we achieve international agreement on terminologies
and definitions used to describe abnormalities of menstrual bleeding? Hum Reprod 2007;22:635–43.

FIGO classification of the causes of AUB

Continuing on the theme of updating AUB terminology, an expert committee


recommended a classification system to describe the causes of AUB in non-
gravid women of reproductive age (Munro et al, 2011). They believed a
comprehensive classification system would facilitate communication between
clinicians, investigators and patients, with the potential to improve patient health
outcomes.

The stimulus for this work was concern that women with AUB were often affected
by more than one component from the ‘PALM-COEIN’ classification. However, it
was not always possible to distinguish whether an individual component was
causal or associated with HMB.

7
FIGO have approved this new classification system and have called it PALM-
COEIN:

 structural causes for AUB: polyp; adenomyosis; leiomyoma; malignancy and


hyperplasia

 non-structural causes for AUB: coagulopathy; ovulatory dysfunction;


endometrial; iatrogenic; and not yet classified.

Using the full notation ‘PALM-COEIN’ it is possible to define women with AUB who
have one or more contributing pathologies. In all cases, the presence or absence
of each criterion is noted using 0 if absent, 1 if present, and ? if not yet assessed.

Reprinted from: Munro MG, Critchley HO, Broder MS, Fraser IS and the FIGO Working Group on Menstrual
Disorders. FIGO classification system (PALM-COEIN) for causes of abnormal uterine bleeding in nongravid
women of reproductive age. Int J Gynaecol Obstet 2011;113:3–13 [Abstract], with permission from Elsevier

In general, the components of the PALM group are discrete (structural) entities
that can be measured visually with imaging techniques and/or histopathology,
whereas the COEIN group is related to entities that are not defined by imaging or
histopathology (non-structural).

The term 'dysfunctional uterine bleeding’ which was previously used as a


diagnosis for AUB that occurs in the absence of systemic or locally definable
genital tract pathology, should be discarded and is not included in PALM-COEIN.
Women who fit this description generally have any combination of coagulopathy,
ovulation or primary endometrial disorder.

The following illustrated table shows how diagnosed pathology can be described
used PALM-COEIN terminology.

8
Reprinted from: Munro MG, Critchley HO, Broder MS, Fraser IS and the FIGO Working Group on Menstrual
Disorders. FIGO classification system (PALM-COEIN) for causes of abnormal uterine bleeding in nongravid
women of reproductive age. Int J Gynaecol Obstet 2011;113:3–13 [Abstract], with permission from Elsevier

The PALM-COEIN system can also include the Type O, 1, 2 submucosal fibroid
classification system (Wamsteker 1993 and ESGE), and expand this further to
intramural and subserosal location of fibroids. This is depicted in the figure
below.

Reprinted from: Munro MG, Critchley HO, Broder MS, Fraser IS and the FIGO Working Group on Menstrual
Disorders. FIGO classification system (PALM-COEIN) for causes of abnormal uterine bleeding in nongravid

9
women of reproductive age. Int J Gynaecol Obstet 2011; 113:3–13 [Abstract], with permission from Elsevier

Type 8 fibroids are leiomyomas that do not relate to the myometrium and include
cervical or broad ligament fibroids without direct attachment to the uterus, as
well as other so-called 'parasitic' (extra-pelvic) lesions.

Munro MG, Critchley HO, Broder MS, Fraser IS and the FIGO Working Group on Menstrual Disorders. FIGO
classification system (PALM-COEIN) for causes of abnormal uterine bleeding in nongravid women of
reproductive age. Int J Gynaecol Obstet 2011;113:3–13 [Abstract].

FIGO classification of the causes of AUB: examples

Assuming pregnancy is excluded, the following are the main causes for AUB in
women of reproductive age group.

Structural
P Endometrial polyps, cervical polyps
A Adenomyosis
L Leiomyoma
M Premalignancy (endometrial hyperplasia)
Malignancy of the genital tract (cervical, endometrial, ovarian, vaginal, vulval,
sarcoma of endometrium or myometrium)
Non-structural
C Systemic coagulopathy, e.g. thrombocytopenia, von Willebrand's disease,
leukaemia, warfarin
O Disorders of ovulatory function, e.g. polycystic ovary syndrome, congenital
adrenal hyperplasia, hypothyroidism, Cushing's disease, hyperprolactinaemia
E Primary endometrial disorders, e.g. disturbances of local endometrial
hemostasis, vasculogenesis or inflammatory response (chronic endometritis)
I Iatrogenic causes, e.g. exogenous sex steroid administration (combined oral
contraceptives, progestins, tamoxifen), intrauterine contraceptive device,
traumatic uterine perforation
N Generally rare causes, e.g. arteriovenous malformations, myometrial
hypertrophy, sex steroid secreting ovarian neoplasm, chronic renal or hepatic
disease, endometriosis

The term ‘dysfunctional uterine bleeding’ which was previously used as a


diagnosis for AUB that occurs in the absence of systemic or locally definable
genital tract pathology, should be discarded and is not included in PALM-COEIN.
Women who fit this description generally have any combination of coagulopathy,
ovulation or primary endometrial disorder.

If there is AUB before menarche then a pelvic examination (usually under


anaesthesia) should be performed. In cases such as these, the differential

10
diagnoses would be: malignancy, trauma, sexual abuse, assault or congenital
malformations.

Key points

Causes of AUB should be classified as per the PALM-COEIN model.

PALM causes (Polyps, Adenomyosis, Leiomyomas [subserosal and other],


Malignancy and hyperplasia) are structural.

COEIN causes (Coagulopathy, Ovulatory dysfunction, Endometrial, Iatogenic,


Not otherwise specified) are non-structural.

Assessment of AUB - generic principles

 Establish if chronic AUB (>6 months) or acute AUB (urgent intervention required).

 Initial step should be to exclude pregnancy.

 Understand the differential diagnosis of AUB (i.e. PALM-COEIN classification) and


accept the concept that women could be affected by more than one component,
but not all components may be causal to AUB.

 Undertake relevant gynaecological history and examination.

 Screen for coagulopathy (such as von Willebrand disease) by using a specific


structured history. If clinical history screen is positive, then undertake
coagulation profile and vWD test.

 Recommendation for full blood count (FBC), cervical smear, pelvic infection
swabs and pelvic ultrasound (and coagulation screen if clinical question screen
positive).

 Referral to secondary care (one-stop or rapid access clinic) if malignancy is


suspected, endometrial biopsy is required, pathology suspected/identified or
medical treatment deemed unsuccessful; collectively termed ‘red flag’ features.

 Most HMB is due to a combination of coagulopathy, ovulatory or endometrial


dysfunction that does not require secondary referral and treatment can be
commenced (and successfully undertaken) in primary care.

 Uterine/endometrial assessment: investigation using hysteroscopy with


endometrial biopsy or ultrasound and endometrial biopsy improves the detection
rate of endometrial pathology (malignant and benign) compared with
hysteroscopy or ultrasound alone.

11
 Adopt hierarchical three-step treatment approach.

What was novel within the 2007 NICE HMB guideline (CG44) was the
recommendation that primary care request a transvaginal pelvic ultrasound scan
as a first-line diagnostic tool for identifying uterine and ovarian pathology, rather
than defer this investigation until a secondary care referral was made.
Transvaginal pelvic ultrasound investigation is indicated if ‘red flag’ features (see
diagram below) suggestive of gynaecological pathology are present.

Furthermore, their identification warrants referral of the woman to the local


hospital one-stop menstrual disorder clinic or rapid access gynaecology clinic.

Outpatient hysteroscopy, endometrial Pipelle biopsy, or colposcopy can be


performed in such clinics to elucidate the precise nature of any uterine or
cervical pathology. Saline infusion sonohysterography and 3D transvaginal pelvic
ultrasound may increase the diagnostic accuracy of detecting gynaecological
pathology, and are an alternative to pelvic ultrasound and
hysteroscopy/endometrial biopsy in selected cases.

12
Overall summary care pathway
PCB: postcoital bleeding; PMB: postmenopausal bleeding; IMB: intermenstrual bleeding;
2ww : urgent appointment within 2-week wait rule for suspected cancer; uterus>10w:
clinical measurements suggests uterus greater 10 weeks size (or uterine cavity >10 cm
length); FBC: full blood count (anaemia tends to indicate severe HMB).
Red flag features: six features are numbered in the figure that, if present,
warrant referral to secondary care.
Adapted from: National Institute for Health and Clinical Excellence. Heavy menstrual
bleeding. NICE clinical guideline 44. London: NICE; 2007.

13
Although several assessment and treatment pathways for AUB have been published,
they essentially portray the same message. Initial assessment in primary care should
include:
1. history: specifically enquiring about menstrual history, pelvic pain, fertility,
symptoms of anaemia and impact on quality of life, risk factors for inherited
coagulopathy (e.g. von Willebrand disease)
2. abdominal and pelvic examination: specifically, a bimanual examination of the
uterus and speculum examination of the cervix, with cervical cytology sampling
undertaken if appropriate
3. genital tract infection screening if risk factors are present or
cervicitis/endometritis is suspected on history or examination.
4. FBC to check for anaemia
5. pelvic ultrasound if ‘red flag’ features (see diagram above) are present.
National Institute for Health and Clinical Excellence. Heavy menstrual bleeding. NICE
clinical guideline 44. London: NICE; 2007.

NICE HMB care pathway

14
History and clinical coagulation screen
1. Define the nature of bleeding:
 age
 menstrual or non-menstrual (intermenstrual bleeding, postcoital bleeding,
postmenopausal bleeding)
 subjective assessment of MBL (sanitary protection usage, flooding, clots,
duration of menstruation) and the woman's personal opinion
 alteration in the menstrual cycle
 pelvic pain and pressure effects
 previous medical or surgical treatment for AUB
 up-to-date smear test
 family history of gynaecological pathology.
2. Identify symptoms that may indicate pathology or need for secondary care
referral (summarized as red flag features).
Pathology may include:
 fibroids (pelvic pain, pelvic mass, pressure/obstructive GI/GUT symptoms)

15
 endometriosis/adenomyosis (cyclical and non-cyclical chronic pelvic pain,
dyspareunia, dysmenorrhoea, infertility)
 inherited or acquire haemostatic/coagulopathy disorder e.g. von Willebrand
disease. See table below. The presence of all three domains on clinical
questioning is highly predictive for a haemostatic condition that could be
contributory/causal to AUB and indicates the need to perform specific
haematological screening blood tests.
 Secondary care referral is indicated if pathology is suspected/identified,
malignancy is suspected and/or failure of medical treatment. Malignancy may
be indicated by: PMB, IMB, PCB, alteration in menstrual cycle, pelvic pain,
pelvic mass, pressure/obstructive GI/GUT symptoms, weight loss/gain.
3. Identify pathological effects such as:
 anemia (request FBC)
 pelvic pain
 impaired quality of life.
4. Identify treatment expectations such as:
 concerns and needs
 future fertility and contraception wishes
 need for definitive treatment when offered treatment alternatives

Structured history to screen for coagulopathies (AUB-C)


Domain Characteristic
1 Heavy menstrual bleeding since menarche
2 One of the following:
postpartum haemorrhage
surgical-related bleeding
bleeding associated with dental work
3 Two or more of the following symptoms:
bruising 1–2 times per month
epistaxis 1–2 times per month
frequent gum bleeding
family history of bleeding symptoms
Table adapted from: Kouides PA, Conard J, Peyvandi F, Lukes A, Kadir R. Hemostasis and
menstruation: appropriate investigation for underlying disorders of hemostasis in
women with excessive menstrual bleeding. Fertil Steril 2005;84:1345–51 [Abstract].

Examination, basic investigations and diagnosis

According to NICE HMB guidelines (2007), if history taking reveals HMB without the
presence of pathology, then there is no need to undertake a physical examination prior
to initiating first-step medical treatment.

This will apply to the vast majority of cases. However, some clinicians have raised
concerns with this approach and it would be reasonable to undertake a clinical
examination particularly if there was:

 uncertainty or overdue routine cervical cancer screening or underlying


pelvic/STI infective process

16
 structural pathology (i.e. PALM part of PALM-COEIN classification): given
severity of symptoms or co-existing pelvic pain and other symptoms such as IMB, PCB
and PMB, weight loss/gain
 suspected coagulopathy: HMB or AUB since menarche, tendency to bruise
easily, and family history of coagulopathy may indicate an inherited or acquired
coagulopathy
 suspected ovulatory dysfunction: Oligoamenorhoea, obesity, acne, hirsutism,
and acanthosis nigricans may be suggestive of polycystic ovary syndrome or diabetes
mellitus, or oligoamenorrhoea and galactorrhoea, which could suggest
hyperprolactinaemia.

Clinical examination

 General examination – pallor (anaemia)


 Abdominal palpation
 Visualisation and palpation of the cervix
 Bimanual (internal) pelvic examination to estimate uterine size and/or adnexal
masses
 If there is identification of fibroids that are intracavitary, or a uterine length
greater than 12 cm with or without suspected uterine fibroids, then referral for
specialist investigation should be made.

Investigations

There is no agreed 'gold standard' as to which methods (or combinations of methods)


of investigation are better at identifying certain types of pathology than others and, as
such, a comprehensive evaluation is required for women with a high risk of benign or
malignant genital tract pathology.

In order to make an assessment more assured, the NICE HMB guidelines has defined
'high risk' women as follows.

Women with AUB who have:

 declined or failed an adequate trial of medical treatment


 aged >45 years
 pathology suspected based on history ± physical examination.

Where AUB includes women with:

 abnormal menstrual bleeding


o HMB with regular and irregular cycles
o HMB and dysmenorrhoea
o ± pelvic pain
o ± pressure effects on GIT/GU tract
 non-menstrual bleeding
o intermenstrual bleeding
o postcoital bleeding
o postmenopausal bleeding.

A comprehensive baseline set of investigations that could be undertaken in


primary care would therefore include:

17
 full blood count
 cervical smear
 pelvic ultrasound
 pelvic infection screening (genital tract swab testing).

The NICE HMB guidelines considers pelvic ultrasound as a useful screening


investigation for any uterine structural anomalies and ovarian pathology. Ultrasound is
more accurate at identifying uterine fibroids than hysteroscopy, whereas hysteroscopy
is better at identifying polyps and endometrial disease.

Commencing treatment in primary care or referring to secondary care

For women with HMB who are under 45 years of age with no obvious pathology based
on any combination of history, physical examination, GP organised investigations (such
as cervical smear, FBC, pelvic ultrasound), there are two options for treatment:

1. immediately refer to secondary care for evaluation


2. commence a 3–6 month trial of medical therapy (e.g. COC, Mirena LNG-IUS) and
if there is no improvement refer the woman to secondary care for evaluation.

Because there is no robust evidence to suggest which is the most effective (clinical and
cost-effective) option, all further treatment should be decided by both the woman and
her GP after due consultation.

National Institute for Health and Clinical Excellence. Heavy menstrual bleeding. CG44.
London: NICE; 2007.

Red flag features of AUB indicating secondary care referral

Primary care is under increasing pressure to reduce the need for referral to specialist
services. HMB lends itself to this goal, as primary care can readily optimise treatment
(particularly by first-line use of Mirena LNG-IUS) and achieve treatment success in the
majority of women within the community setting.

There are likely to be numerous locally produced, integrated primary and secondary
care pathways for HMB that focus on improving patient outcomes; an aspiration set out
by recent NHS reforms.

However, a key fundamental goal of primary care is to appreciate when to refer women
presenting with AUB for secondary care assessment (either one-stop ambulatory
menstrual disorder clinic or rapid access gynaecology clinic). Such 'red flag’ features of
AUB that warrant referral to secondary care are listed in the table below.

Pathology suspected before treatment


1 Suspected gynaecological cancer PCB
PMB
IMB
pelvic mass
cervix lesion

18
2 Requires endometrial biopsy (to rule Persistent IMB
out endometrial hyperplasia or >45 years with treatment failure
endometrial malignancy) Irregular bleeding while on hormone-
replacement therapy or tamoxifen
Pathology identified before/after treatment
3 Enlarged uterus (clinically measures Fibroids, adenomyosis
>10 weeks size or >10 cm uterine
cavity length on uterine sounding)
4 Moderate/severe anaemia on FBC Usually benign pathology such as fibroids,
endometriosis
Uterine/ovarian pathology identified on
pelvic ultrasound scan
Identification of
coagulation/haemostatic disorder on e.g. von Willebrand disease
clinical screening and testing

Pathology suspected after treatment


5 Medical treatment has failed At least 3 months of drug treatment (at least
6 months of Mirena) and failure is based on
woman’s own assessment
6 Patient wishes for surgery Endometrial ablation, hysterectomy

Uterine and endometrial assessment

Assessment of the uterus and endometrium is conducted in women at high risk of


benign or malignant genital tract pathology. This essentially comprises of three
components:

1. pelvic ultrasound
2. hysteroscopy
3. endometrial biopsy; either hysteroscopically directed or through 'blind' global
uterine cavity Pipelle® sampling.

In addition to new AUB terminology and classification of AUB causes (PALM-COEIN),


FIGO also recommended an uterine/endometrial assessment care pathway (depicted
below).

19
Reprinted from: Munro MG, Critchley HO, Broder MS, Fraser IS and the FIGO Working
Group on Menstrual Disorders. FIGO classification system (PALM-COEIN) for causes of
abnormal uterine bleeding in nongravid women of reproductive age. Int J Gynaecol
Obstet 2011;113:3–13 [Abstract], with permission from Elsevier
Enlarge

Saline hysteroscopy

Hysteroscopy combined with endometrial biopsy improves the sensitivity and


specificity for detection of endometrial malignancy and other pathology compared with
either diagnostic tests performed alone.

The endometrial biopsy may either be taken under direct hysteroscopic vision (e.g.
using the 5 French operating channel of the hysteroscope and inserting a hysteroscopic
grasper or biopsy forceps) or in a non-directed 'blind' manner sampling all surfaces of
the uterine cavity e.g. Pipelle® sampler.

20
Hysteroscopy has a sensitivity and specificity for identifying endometrial cancer of 86%
and 99%, respectively. Endometrial sampling alone has a sensitivity and specificity of
68–81% and 99–100% for identifying endometrial hyperplasia and endometrial cancer.

Saline infusion sonography

2D transvaginal pelvic ultrasound images of A) normal uterus and right ovay and B) a
normal uterus.

Saline infusion sonography (SIS) provides improved visualisation of uterine and


endometrial pathology. SIS involves an infusion of sterile saline through a soft, plastic
catheter placed in the cervix in conjunction with transvaginal ultrasound. The saline
infusion distends the uterine cavity.

Enlarge

21
SIS outlines the intrauterine polyp (Figure B), which could not be easily seen with in the
routine transvaginal pelvic ultrasound (Figure A).

Supplementary investigations
Consideration may be given to other investigations, which may be requested from both
primary and secondary care settings.
These are summarised in the table below.
Investigations supplementary to baseline set
MRI and endometrial Particularly for assessing suitability for UAE or surgical
biopsy therapy for fibroids
Particularly for advanced stage endometriosis or features
suggestive of endometriosis recurrence
Sonohysterography Routine pelvic ultrasound identifies more uterine fibroids
(saline infusion than hysteroscopy but fewer polyps. Ultrasound will also
sonography) assess for ovarian pathology
Pelvic infection Particularly reproductive age groups or where sequelae of
screening STI (sexually transmitted infection) are suspected
Von Willebrand's Around 5–20% of women with menorrhagia have an inherited
disease (vWD) bleeding disorder, most often vWD
Indications to test for vWD include: menorrhagia since
menarche, family history of idiopathic menorrhagia, easy
bruising and/or personal history of easy bruising or dental
bleeding

Hysteroscopy
Assessment history:
Score
Total attempts: 0
Average score: 0%

For each complication listed below, choose the frequency of their occurrence when
undergoing diagnostic or operative hysteroscopy. Each option can be used once, more
than once or not at all.

A: < 1% frequency

22
B: < 5% frequency
C: 5–10% frequency

Cervical laceration.

Please Select

Cervical and/or uterine false passage if inserting hysteroscope under direct


vision.

Please Select

Uterine perforation +/- damage to abdominal viscera (bowel/bladder).

Please Select

Primary haemorrhage.

Please Select

Fluid overload and dilutional hyponatraemia (particularly relevant with first-


generation endometrial destruction techniques that used Glycine; now we
use bipolar resectoscopes that work in saline uterine distension media).

Please Select

Electrosurgical injury.

Please Select

Secondary haemorrhage.

Please Select

Postoperative infection.

Please Select

Hysterectomy (due to bleeding, perforation).

Please Select

23
Vasovagal syncope.

Please Select

Vaginal bleeding and discharge.

Please Select

Pain: pelvic or shoulder.

Please Select

Postmenopausal bleeding
Unscheduled bleeding in postmenopausal women is abnormal and may indicate the
presence of endometrial cancer.
Nearly all cases of endometrial cancer (96%) are associated with a thickened
endometrium (>4 mm in postmenopausal women), which can be measured by
transvaginal ultrasonography. If the scan shows a thickened endometrium >4 mm, then
histological sampling is essential in the diagnostic evaluation of abnormal bleeding in
postmenopausal women.
Hysteroscopy combined with endometrial biopsy improves the detection of intrauterine
pathology and has a high specificity (highly unlikely not to detect cancer). SIS is a
sensitive technique for the detection of intrauterine, intramural and ovarian pathology
but does not provide histological data.
Causes of vaginal bleeding in postmenopausal women
Polyps 30%
Submucosal fibroids 20%
Endometrial atrophy 30%
Hyperplasia 8–15%
Endometrial carcinoma 8–10%
Ovarian, tubal, cervical malignancy 2%

PMB, HRT, tamoxifen and endometrial cancer

More than 90% of the cases of endometrial cancer occur in women over 50 and this
cancer is associated with 10% of the cases of postmenopausal vaginal bleeding.

To this end, any unscheduled bleeding while on HRT should be referred to secondary
care for further investigation. Manufacturers of HRT have advised that erratic bleeding
may occur in the intial 3 months of HRT commencement, or when changing
preparations. However, unscheduled bleeding persisting beyond this period warrants
urgent referral and investigation.

24
Tamoxifen usage, like HRT, also increases the risk of developing endometrial
hyperplasia or cancer, but TVUS can be misleading in these patients.

Tamoxifen can cause subendometrial cyst development, which makes the endometrium
appear thickened in transvaginal sonograms. However, the subendometrial cystic
tissue can be differentiated from the endometrium itself in SIS.

Endometrial thickness

The thickness of the endometrial stripe can be measured accurately by transvaginal


sonography and it has been estimated that 96% of postmenopausal women with
endometrial cancer will have an endometrial thickness (ET) >4 mm. At this threshold,
the false positive rate is 50%.

Women with PMB whose ET is <4 mm still have a 1–2% risk of having endometrial
cancer. TVUS can also show if the endometrial lining is very thin. If so, the bleeding
may be due to endometrial atrophy.

It has, therefore, been suggested that as a minimum screening test an endometrial


biopsy is required if ET is:

 >4 mm in postmenopausal women


 >16 mm in premenopausal women
 and may be selectively performed in postmenopausal women with ET <4 mm if
other historical, clinical or sonographic risk factors are present.

However, endometrial biopsy may fail to provide sufficient tissue for histology and is
reported to have up to an 11% false negative rate for the detection of endometrial
cancer.

25
Testing for endometrial cancer

Endometrial cancer
The preferred investigation of endometrial cancer is hysteroscopy and either blind
endometrial biopsy (using Pipelle sampler) or hysteroscopic-guided endometrial biopsy.
This diagnostic test may be expanded in a 'therapeutic manner' by performing
hysteroscopic resection of any identified intrauterine focal lesions (polyps, submucous
fibroids), i.e. see-and-treat.
This test has a 99% specificity in women with PMB.
If this test is negative, then endometrial cancer is highly unlikely as the post-test
probability of endometrial cancer is <0.5%.
Predictive values for endometrial cancer in postmenopausal women
Sensitivity Specificity PPV NPV
Transvaginal ultrasound (TVU) 67% 56% 7% 97%
Endometrial biopsy (EMB) (blind)* 87% 98.5% 82% 99.1%
SIS 89% 46% 16% 97%
Hysteroscopy and biopsy 86% 99.2% 100% 99.5%
PPV = positive predictive values.
NPV = negative predictive values.

Case study
A 56-year-old woman presents with two episodes of PMB. She has taken continuous
combined HRT for the last 3 years. Her last cervical smear was 2 years ago and was
normal.
Below is an image of her transvaginal pelvic ultrasound investigation, which shows her
endometrial thickness being 14 mm (1.48 cm).

26
Based on the patient history and investigations, what assessments would you
perform next?

Write your answer in the reflective notes before proceeding to the next page.

Assessment

The following key assessments follow a investigation of the woman's clinical history:

 systemic examination: examine for lymphadenopathy, anaemia, breast


examination (screen for primary malignancy in the breast, which could present as
secondary metastases in the genital tract)
 abdominal examination: any organomegaly, pelvic masses
 pelvic examination: inspection of the vulva, vagina and cervix for any obvious
neoplasia. Palpate the cervix and assess uterine size and mobility.

Key investigations

 Cervical smear (consider even if recently shown to be normal on national


surveillance strategy).
Atypical glandular cells of endometrial cell origin on a cervical smear indicate urgent
referral for hysteroscopic evaluation as uterine cancer, endometrial hyperplasia or
cervical adenocarcinoma are present in 30–40% of such cases.
 Transvaginal pelvic ultrasound.
 Depending on the pelvic ultrasound:
o hysteroscopy and endometrial biopsy
o ovarian tumour markers (CA125, CEA, CA-19-9) if ovarian neoplasm is
suspected.
PMB
Assessment history:
Score
Total attempts: 0
Average score: 0%
Answer whether the following statements are true or false.

Tamoxifen can cause sub-endometrial cyst development that may be

27
detectable by SIS

True

False

Endometrial cancer is highly improbable if endometrial thickness is <4 mm in


a woman presenting with PMB

True

False

SIS is more sensitive than TVUS for detecting focal abnormalities of the
endometrium

True

False

Hysteroscopy is more sensitive than SIS for uterine polyp detection

True

False

Endometrial cancer is present in 40–60% of cases of PMB

True

False

Intermenstrual bleeding

Intermenstrual bleeding (IMB) encompasses any bleeding which occurs outside of the
woman’s menstrual period. It may affect between 13–21% of women. The incidence in
perimenopausal women is as high as 24%. IMB can also include postcoital bleeding. It is
a symptom which although largely attributed to a benign cause, can result in significant
distress to the patient as it is a commonly known and accepted symptom of cervical
cancer.

28
Causes

Causes of IMB can be classified based on their anatomical site. Commencing at the top
of the reproductive tract they are:

 Ovarian causes: 1–2% of women will spot at ovulation. Estrogen secreting


ovarian tumours can also cause IMB in postmenopausal women.
 Uterine causes:
o iatrogenic – irregular bleeding secondary to hormonal contraceptives
(COC, POP, IUS [Mirena]) and hormonal implants. The bleeding may be due to the
medication itself, its misuse e.g missed pills, or interactions with other medications like
enzyme inducers. Other medications include drugs which affect the clotting pathway
e.g. SSRI’s and anticoagulants.
o infective – endometritis
o structural benign – uterine polyps and fibroids, adenomyosis
o structural malignant – endometrial cancer.
 Cervical causes:
o iatrogenic – following examination/smear test
o infective – cervicitis secondary to infection usually chlamydia or
gonorrhoea
o structural benign – cervical ectropion can occur spontaneously or in
response to increased estrogen levels secondary to pregnancy or the combined oral
contraceptive pill. Cervical polyps 1.5–10% prevelance, largely benign (malignancy
0.1% and dysplasia 0.5%)
o structural malignant – cervical cancer; the estimated prevalence of CIN
and cervical cancer is 3–18% in women with PCB.
 Vaginal causes:
o infective – chlamydia and gonorrhoea may cause cervicitis as above.
Vulvovaginitis secondary to Trichomonas vaginalis or Candida albicans infection may
present with IMB in the context of severe vaginal/vulval oedema or excoriations.
o structural benign – adenosis (metaplastic cervical or endometrial tissue on
the vaginal wall)
o structural malignant – vaginal cancer.

Examination and investigation

As outlined in the AUB bleeding section, thorough clinical history taking and
examination is vital to elicit and identify the possible causes of IMB. Undertaking
speculum examination should allow identification of any cervical or vaginal
abnormalities. A cervical smear should be taken only if the patient is due one based on
national recall (3 yearly for women aged 25–49 and 5 yearly from 49–64).

Referral to colposcopy should be made if there is an obvious abnormality of the cervix


or if there are symptoms of cervical cancer (persistent postcoital bleeding or persistent
vaginal discharge which cannot be explained by other causes such as infection, polyp
etc). The StratOG core training tutorial on Pre-invasive disease of the lower genital
tract has further information on colposcopy.

29
Cervical cytology with high vaginal and endocervical swabs should be obtained for the
diagnosis of STI’s. See the StratOG core training tutorial on Sexually transmitted
infections (including HIV) for detailed information on the investigation and
management of STI’s.

Causes Investigation Treatment


Infection Bacterial/viral Appropriate
Endometritis swabs +/- antibiotics/antivirals/antifungals
Cervicitis imaging
Vulvovaginitis
Iatrogenic As clinically Alternating dosage/type of
Breakthrough bleeding indicated hormone administered +/-
Secondary to tranexamic acid
examination/smear test.
Structural (benign) Radiological Treat the cause in accordance
Uterine/cervical imaging +/- with the woman’s wishes and
polyps/fibroids hysteroscopy local resources
Ectropion
Vaginal adenosis
Structural In accordance In accordance with local and
(premalignant/malignant) with local and national protocols
Uterine/cervical/vaginal/v national
ulval cancer protocols
CIN/VaiN/VIN
Ovarian estrogen
secreting tumors
Natural If clinically Reassurance
1–2% of women will have indicated
midcycle spotting, associated
with ovulation

NHS Cervical Screening Programme. Colposcopy and Programme Management;


Guidelines for the NHS Cervical Screening Programme. 3rd ed. NHSCSP Publication No
20. Sheffield: NHSCSP; 2016.

Wan LY, Edmondson RJ, Crosbie EJ. Intermenstrual and postcoital bleeding. Obstet
Gynaecol Reprod Med 2015;25:106–112.

Rao S. Intermenstrual and post-coital bleeding. Obstet Gynaecol Reprod Med


2011;21:288–291 [Abstract].

IMB case study

A 34-year-old woman presents with a 6-month history of intermenstrual bleeding,


prolonged menses (6–12 days) and dysmenorrhoea.

She is experiencing unscheduled bleeding while on a combined oral contraceptive

30
(COC). She has been taking a COC for 6 years. Her last cervical smear (performed 3
months ago) was normal, as was her clinical pelvic examination.

The consultant refers the woman to secondary care and a transvaginal pelvic
ultrasound is requested.

The report states:

 uterine cavity 7 cm
 normal ovaries (left 2 cm, right 3 cm with follicle)
 thickened endometrium (12 mm) with a prominent endometrial echo (25 mm by
20 mm) indicating a possible polyp.

The following management options are considered:

Option 1: saline infusion sonography.


Option 2: outpatient diagnostic hysteroscopy.

Option 1: saline infusion sonography

Sonohysterography revealed an endometrial polyp originating


from the posterior wall of the uterine fundus

31
Color Doppler imaging demonstrates a vascular stalk (arrow)

Option 2: outpatient diagnostic hysteroscopy

An intrauterine polyp (3 x 2 cm) located at the fundus was identified through outpatient
diagnostic hysteroscopy.

Based on the patient history and investigations, what treatment option would
you suggest?

Write your answer in the reflective notes before proceeding to the next page.

IMB case study answer

Based on the patient history and investigations, what treatment option


would you suggest?

Answer: Hysteroscopic resection is a highly effective therapeutic option. The polyp in


this case was resected hysteroscopically in the outpatient setting using Versapoint®.

Key points
Uterine polyps, whether endometrial or submucous fibroid polyps, are a
common cause of intermenstrual bleeding (IMB).
Once diagnosed by hysteroscopy, uterine polyps are easily treated by
hysteroscopic techniques with highly successful results in the vast majority of women.
1477756374 frmReflectiveNote

Causes of IMB

32
Assessment history:
Score
Total attempts: 0
Average score: 0%

Are the following recognised causes of intermenstrual bleeding (IMB)?

Answer whether the following statements are true or false.

Thrombocytopenia

True

False

Sex steroid secreting ovarian neoplasm

True

False

Endometrial polyp

True

False

HRT

True

False

Intrauterine device

True

False

Etonogestrel implant

33
True

False

Treatment of HMB - a three-step approach


In the early 1990s, it was estimated that at least 60% of women presenting with HMB
would have a hysterectomy as the first-line treatment. The majority of these
hysterectomies occurred in women without uterine pathology.
The number of hysterectomies is now decreasing. This decline can be partially
attributed to the implementation of the NICE HMB three-step treatment hierarchy and
increased uptake of Mirena LNG-IUS and endometrial ablation treatments.
NICE three-step treatment approach
The NICE HMB clinical guideline (2007) recommends a three-step hierarchal approach
to determining treatment options. The table below utilises this classification and has
updated the evidence base for available treatments.
The guideline has stated that while Mirena LNG-IUS is preferable to other medical
treatments (tranexamic acid, NSAIDs, COC), this recommendation is based on indirect
evidence.
In a more recent multi-centre randomised controlled trial (ECLIPSE) of 571 women
presenting in the primary care setting with HMB, LNG-IUS (Levonorgestrel intrauterine
system) was directly compared to usual medical treatments (tranexamic acid,
mefenemic acid, combined estrogen and progesterone or progesterone alone therapy).
When assessed over a 2-year period, LNG-IUS was more effective than usual medical
treatments in improving heavy menstrual bleeding. More women were still using the
LNG-IUS at the end of 2 years in view of its beneficial effects. In addition, secondary
outcomes such as practical difficulties, family life, work and daily routine, psychological
wellbeing were significantly better in women who received treatment with the LNG-IUS.
There were no significant between-group differences in rates of surgical intervention or
sexual activity scores. An intention to treat analysis is planned in 5 and 10 years.
For women with HMB and large fibroids and/or significant symptoms, such as chronic
pelvic pain or dysmenorrhoea (pressure symptoms), a hysterectomy or uterine artery
embolisation may be offered as first-step therapy. Similarly, women with significant
diagnoses like endometriosis can also be offered hysterectomies as primary treatments.
Although hysterectomy is a major operation associated with significant morbidity in a
minority of cases, it offers a definitive cure and achieves the highest satisfaction rates
in the long term.
Treatment Effect on fibroid Effect on HMB Effect on fertility
size
First-step treatments (all medical)
Tranexamic acid No effect Decrease 30–50% No effect
NSAIDs (e.g. mefenamic No effect Decrease 20–40% No effect
acid)
Combined oral No data Decrease 40% Licensed
contraceptive (synthetic contraceptive
estrogen)
Combined oral No data Decrease 30% Licensed
contraceptive Qlaira contraceptive

34
(natural estrogen)
Oral progestogen (high- No effect Decrease 60% Contraceptive effect
dose) or licensed
contraception
Intrauterine Decrease 30% Decrease 70–100% Licensed
progestogen (LNG-IUS) (may also treat contraceptive
endometriosis and
adenomyosis)
GnRH analogues (3–6 Decrease 30% Decrease 60–100% Likely contraceptive
months with/without (causes but contraception is
add-back HRT) amenorrhoea in 80– advised
90% of women)
Progestogen-only Unknown Decrease 30–100% Licensed
implant (Nexplanon) or (causes contraceptive
Progestogen-only amenorrhoea in 15–
injectable 20%)
(Depoprovera)
Second-step treatment (minimally invasive uterus-conserving surgery)
Hysteroscopic Excision and Decrease 50–80% Improved if excising
myomectomy removal of submucous fibroid
(hysteroscopic fibroid intracavitary No effect if excising
resection) fibroids uterine polyp
Endometrial ablation No effect Decrease 80% (may Likely contraceptive
also treat but contraception is
adenomyosis) advised following
ablation
Transcervical resection Will be able to Decrease 80%-100% Likely contraceptive
of endometrium excise and but contraception is
removal advised following
intracavity fibroids ablation
Laparoscopic Excise subserosal No effect or may No effect or may
myomectomy and non-deeply decrease up to 30% increase
embedded
intramural fibroids
Second-step OR third-step treatment (established and newly developed
minimally invasive uterus-conserving treatments)
Focal fibroid treatment
MRI-guided focused Decrease 15–20% Decrease 60% May decrease, have
ultrasound therapy no effect, or
improve fertility
Radiofrequency ablation Decrease 50-80% Decrease 50% Treatment still
of fibroids under trial
(hysteroscopic system is
VizAblate
Global uterus treatment
Uterine fibroid Decrease 30% Decrease 60–80% May decrease, have
embolisation no effect, or

35
improve fertility
Laparoscopic uterine Decrease 20-30% Decrease 50%-60% Treatment still
artery occlusion (with or under trial
without ovarian artery
occlusion)
Doppler-guided Decrease 20-30% Decrease 40-60% Treatment still
transvaginal uterine under trial
artery occlusion
Third-step treatments (major surgical procedures)
Abdominal Excise subserosal, Decrease 60–80% Improved,
myomectomy intramural and particularly if
intracavitary uterine cavity is no
lesions longer distorted
Hysterectomy Complete cure Complete cure Irreversible
contraceptive
National Institute for Health and Clinical Excellence. Heavy menstrual bleeding. CG44.
London: NICE; 2007.
National Institute for Health and Clinical Excellence. Heavy menstrual bleeding: quick
reference guide. CG44. London: NICE; 2007.
Gupta J, Kai J, Middleton L, Pattison H, Gray R, Daniels J, et al. Levonorgestrel
intrauterine system versus medical therapy for menorrhagia. N Engl J Med
2013;368:128-37.

NICE guidelines for HMB treatment


First-step medical treatment
For NICE first-step medical treatments for HMB, see Table 1 'Pharmaceutical treatments
proven to reduce menstrual bleeding' on page 7 of the NICE quick reference guide,
CG44 (to change the view of a pdf, you can right-click and select 'Rotate Clockwise', or
select the options in the Adobe menu, 'View/Rotate View/Clockwise').
Second- and third-step medical treatment
For NICE second- and third-step treatments for HMB, see Table 2, 'Surgical and
radiological treatment options for women whose quality of life is severely impacted' on
page 8–9 of the NICE quick reference guide, CG44.
National Institute for Health and Clinical Excellence. Heavy menstrual bleeding: quick
reference guide. CG44. London: NICE; 2007.

Levonorgestrel-releasing intrauterine system (LNG-IUS)

36
The levonorgestrel intrauterine-
releasing system (LNG-IUS) is an intrauterine, long-term progestogen-only method of
contraception currently licensed for 5 years of use.
It has a T-shaped plastic frame with a rate-limiting membrane on the vertical stem,
releasing a daily intrauterine dose of 20 microgram/24 hours of levonorgestrel with little
systemic absorption.
The LNG-IUS inhibits endometrial proliferation, thickens cervical mucus and suppresses
ovulation. The licensed and non-licensed uses are indicated in the table below.
Licensed in UK Unlicensed in UK
Contraception Dysmenorrhoea
(5-year duration) Uterine protection for women
receiving tamoxifen therapy
Menorrhagia-DUB Endometriosis, adenomyosis
Fibroids
Uterine protection with estrogen replacement Endometrial hyperplasia
therapy in perimenopausal and postmenopausal Endometrial cancer, if unfit for
women primary surgery
The main side effect, often cited as the reason for discontinuation, is erratic spotting.
This tends to subside 3–6 months from insertion. After 1 year of usage, there is a 71–
95% reduction in objectively measured MBL and around 50% women have
amenorrhoea.
Indirect comparison has shown that LNG-IUS generates more quality-adjusted life years
(QALY) than other medical treatments (tranexamic acid, NSAIDs, COCP) and at a lower
cost. Therefore, LNG-IUS is the recommended first-line treatment for HMB.
In relation to second-line treatments LNG-IUS produces similar satisfaction rates as
endometrial ablation and hysterectomy. A randomised controlled trial compared Mirena
with no treatment in women awaiting hysterectomy for menorrhagia and showed the
following:
Mirena No treatment
At 1 year 68% continued with Mirena 32% had
hysterectomy

37
At 5 years 58% continued with Mirena 42% had
hysterectomy
Cost–benefit Mirena was 40% cheaper than
analysis: hysterectomy

1477756708 frmReflectiveNote

Endometrial resection and endometrial ablation

There are several minimally invasive procedures to destroy the endometrium in


premenopausal women with menorrhagia due to benign causes for whom childbearing
is complete.

Traditionally, first-generation techniques involved resection (transcervical resection of


the endometrium [TCRE]) or ablation (rollerball) of the endometrium under direct
hysteroscopic vision using electrocautery (either monopolar or bipolar).

First-generation techniques required experienced operators and were associated with


complications dependent on both operator experience (e.g. uterine perforation) and the
method itself (e.g. dilutional hyponatraemia if using glycine uterine distension in
monopolar diathermy TCRE).

A) Transcervical resection of the endometrium, B) Hysteroscopic rollerball ablation

Second-generation techniques, compared with those of the first, are non-resectoscopic


(not performed under hysteroscopic vision), only ablate the endometrium, are easier to
learn and perform and have lower rates of serious complications.

Randomised controlled trials comparing first- and second-generation techniques have


found no significant differences in terms of amenorrhoea, bleeding patterns,
premenstrual symptoms, patient satisfaction or quality of life. However, all second-
generation ablation techniques have significantly shorter operating and theatre times
than first-generation techniques and incurred fewer perioperative adverse effects.

Short-term effectiveness (up to 2 years) is high for first- and second-generation


endometrial destruction: around 25–35% of women treated experience amenorrhoea,
with over 90% experiencing satisfactory reduction in menstrual blood loss without the
need for further treatment.

38
Furthermore, 95% of women return to normal activities by 2 weeks. However, upon
longer follow up (around 5 years), around 20–30% of women opting for ablation will
either be dissatisfied or require secondary treatment (usually hysterectomy).

Second-generation endometrial ablation techniques


 Thermal balloon endometrial ablation (Thermachoice®, Cavaterm®)
 Impedance bipolar radiofrequency ablation (NovaSure®)
 Hydrothermablation (HydroThermAblator®)
 Endometrial cryotherapy (Her Option®).

Thermal balloon endometrial ablation (ThermaChoice). Gynecare ThermaChoice uterine


balloon therapy system uses a balloon filled with heated sugar (dextrose) solution
© GYNECARE, a division of ETHICON INC., Somerville, N.J.

NovaSure impedance controlled endometrial ablation system delivers electrical current


to a triangular metallic mesh electrode to vaporise the endometrium
© Novacept, Inc., Palo Alto, California

39
A) Hydro ThermAblator uses heated salt water (saline solution)
© BEI Medical Systems Inc., Teterboro, N.J.,
B) Her Option uterine cryoblation therapy system uses a cryosurgical probe to apply
extreme cold
© CryoGen, Inc., San Diego

All techniques are performed as day-case procedures with some being able to be
performed under local anaesthetic (with or without sedation) in an outpatient setting.

There is evidence to suggest that benefits result from thinning the endometrium (with
medical therapy such as GnRHa) prior to ablation, or performing the ablation just after
menses is complete. However, most techniques have now been shown to be equally
effective without such additional precautions.

Uterus-conserving endometrial ablation treatment may be offered if:

 HMB is severe enough to adversely impact on a woman's quality of life


 the woman has completed her family
 her uterine cavity is normal sized (uterine cavity length <10 cm) with benign
histology.

Normally, ablation is offered as a second-step treatment after previous medical


treatments have failed. However, it could also be offered as a first-step treatment after
a full discussion of outcomes and other treatment options.

Endometrial ablation should only be considered in women who do not wish future
fertility, who have a normal-sized uterus (10 cm uterine cavity length or less), and have
either no fibroids or submucous/intramural fibroids no larger than 3 cm in size.

Women suitable for endometrial ablation

Endometrial ablation (uterus-conserving) and hysterectomy are treatment options


offered to women with unsuccessful medical treatment (including Mirena LNG-IUS) for
their menorrhagia. Informed consent is required that provides the individualised risks
and benefits of the treatment against other potential alternatives.

However, endometrial ablation is best suited to women whose:

 uterus is not greater than 10 cm in size (although microwave ablation may be


used in uteri up to 14 cm cavity length)
 uterus does not contain large fibroids that may be distorting the uterine cavity
 intrauterine cavity does not contain any polyps or fibroids greater than 3 cm,
otherwise these should be removed prior to ablation
 uterus has not undergone any previous endometrial ablation procedure, active
infective process and whose myometrium is at least 10 mm if using the microwave
ablation technique

40
 family is complete or they have no desire for future fertility as decreased
fecundity as well as the risk (mainly theoretical) of harm to mother and fetus during
pregnancy follows ablation.

Recommendations prior to endometrial ablation


1. An endometrial biopsy is obtained and histologically analysed to exclude the
possibility of endometrial hyperplasia or endometrial cancer.
2. Hysteroscopy is performed immediately prior to the insertion of the ablation
device to ensure that:
 any sounding or dilation of the cervix has not caused a perforation or false
passage, resulting in subsequent introduction of the ablation device into the wrong
space
 there is no significant intrauterine pathology that would preclude ablation, e.g.
uterine cavity distorted by intrauterine fibroids
 the operator can accurately determine uterine cavity length (and cervical canal
length if using NovaSure®).

Endometrial ablation
Assessment history:
Score
Total attempts: 0
Average score: 0%

Regarding endometrial ablation...

Answer whether the following statements are true or false.

Second-generation ablation techniques are less costly than hysterectomy


but hysterectomy provides greater gain in quality of life over the long term

True

False

Hysteroscopy should be undertaken post-dilation of the cervix and prior to


insertion of the ablation device in the uterus to confirm uterine integrity

True

False

Endometrial thinning pre-ablation is required for most ablative techniques

41
True

False

Endometrial ablation provides effective contraception

True

False

Hysteroscopic sterilisation may be conducted at the same time as


endometrial ablation

True

False

Endometrial ablative techniques should be undertaken under local


anaesthetic where appropriate

True

False

Hysterectomy

Vaginal hysterectomy

Individual patient assessment is essential when deciding the route of hysterectomy,


and the following factors need to be taken into account:

 presence of other gynaecological conditions or disease


 uterine size
 presence and size of uterine fibroids
 mobility and descent of uterus
 size and shape of vagina
 history of previous surgery.

There is insufficient evidence on the safety and role of total laparoscopic hysterectomy
(NICE guidance). However, laparoscopic-assisted vaginal hysterectomy may be offered

42
to the woman provided she has been informed of the alternatives and their
risks/benefits.

Abdominal hysterectomy

Preoperative GnRHa treatment for 3–4 months may facilitate surgery by reducing
preoperative anaemia, intraoperative blood loss and the need for transfusion. Studies
have shown a reduction of fibroid size of by 60%, which may enable a lower transverse
incision (lower morbidity) rather than vertical midline laparotomy incision (higher
morbidity) when conducting the abdominal hysterectomy.

Indications

Hysterectomy should be only considered where:

 other treatment options have failed or are inappropriate


 women have completed their families
 there is a wish for amenorrhoea
 women (who have been fully counselled) request it or other forms of further
treatment are contraindicated.

The five main routes of hysterectomy are:


1. Abdominal hysterectomy (AH)
2. Vaginal hysterectomy (VH)
3. Laparoscopic hysterectomy (LH) which includes:
laparoscopic-assisted vaginal hysterectomy (LAVH). LAVH is a vaginal
hysterectomy assisted by laparoscopic procedures that do not include uterine artery
ligation
laparoscopic hysterectomy [LH(a)]. LH(a) are laparoscopic procedures, which
include uterine artery ligation

4. Total laparoscopic hysterectomy (TLH)

In TLH there is no vaginal component and the vaginal vault is sutured


laparoscopically
5. Subtotal hysterectomy (STH)

STH (conservation of the cervix) compared with total AH

 After STH, there is a 2–7% risk of persisting cyclical bleeding, 2% risk of cervical
prolapse and a 1% risk of cervical cancer.
 There is no difference between STH and AH in terms of quality of life,
constipation, prolapse, satisfaction with sex life, pelvic pain, vaginal bleeding or
complication rates.

Factors used to determine the method of hysterectomy

43
The decision about the appropriate route for hysterectomy depends on:

 the size of the uterus


 the size of any uterine fibroids (large uterus and/or uterine fibroids make it more
difficult to use less invasive techniques)
 the location of any uterine fibroids (their size and presence increases the risk of
vascular, ureteric or bladder injury)
 the mobility of the uterus
 the size and shape of the vagina
 operator experience
 patient preference
 history of previous surgery
 the evidence based comparison of the methods: first line, vaginal; second line,
abdominal; and third line, laparoscopic.

Evidence based comparison of routes of hysterectomy

A systematic review by Johnson et al has assessed the most appropriate surgical route
for hysterectomy in women with benign gynaecological conditions.

The review concluded that the route of hysterectomy should be in the following order of
clinical preference:

 first line – vaginal


 second line – abdominal
 third line – laparoscopic.

Furthermore, this review established that:

 if possible, VH should be performed in preference to AH because of the rapid


recovery time and fewer incidences of febrile episodes postoperatively
 depending on the uterine descent in the vagina and operator experience, a large
multifibroid uterus can still be removed as a VH rather than AH
 if VH is not possible, then LH has some advantages over AH (decreased operative
blood loss, rapid recovery, fewer febrile episodes and lower wound infection rates) but
at the expense of an increased risk of urinary tract (bladder or ureter) injury and
possibly operating time.

Complications of hysterectomy

Complication AH VH LAVH, LH, TLH


Death 0.5 per 1000
Major perioperative complications 4.5% 4.3% 7.8% (6.1% is
intraoperative)
Severe urinary incontinence* –5% to +20%*
Severe nocturia* –5% to +10%*
Blood transfusion 3–4%*
Bowel injury 0.7% <0.01 0.2%

44
%
Vascular injury 0.8% 0.9% 1.8%
Pelvic haematoma 4–6% 4–6% 4–6%
Vaginal vault infection 2% 2% 4%
Abdominal wound infection 7% 0% 2%
Conversion to laparotomy NA 3% 4%
Urinary tract injury (bladder or urethral) 0.8% 1.6% 2.3%
Bleeding 1.6% <0.01 0.4%
%
UTI 5% 1.3% 5%
Chest infection 5% 7% 0.6%
Other febrile condition 13% 7% 10%
Venous thromboembolism <0.01 <0.01 0.6%
% %

Table adapted from NICE HMB guideline (2007).

*A negative percentage (i.e. -5%) indicates that hysterectomy was associated with a
5% improvement in urinary incontinence and nocturia as well as a 10–20% worsening
of these symptoms.

Johnson N, Barlow D, Lethaby A, Tavender E, Curr L, Garry R. Methods of hysterectomy:


systematic review and meta-analysis of randomised controlled trials. BMJ
2005;330:1478–86.

National Institute for Health and Clinical Excellence. Heavy menstrual bleeding. CG44.
London: NICE; 2007.

Hysterectomy
Assessment history:
Score
Total attempts: 0
Average score: 0%
Answer whether the following statements are true or false.

The number of hysterectomies performed for benign gynaecological


disorders decreased by over 50% between 1993–2002

True

45
False

Approximately 85% of women are willing to accept a 50:50 chance of


treatment failure for HMB in order to avoid hysterectomy

True

False

There is no significant difference in urinary tract injury (includes bladder and


ureter) between laparoscopic hysterectomy and abdominal hysterectomy

True

False

There is no statistically significant differences in urinary tract injury for LH


versus VH or for LH(a) versus LAVH

True

False

There are fewer abdominal wall infections and other postoperative infective
episodes for LH versus AH

True

False

In women undergoing VH compared with AH, hospital stay was shorter and
there was a quicker return to normal activities

True

False

The presence of fibroids at a hysterectomy increases the risk of operative


complications

46
True

False

There is no medical advantage to perform medical pre-treatment before


hysterectomy and myomectomy with GnRH-a for 3–4 months for women with
enlarged uterine fibroids

True

False

The route of hysterectomy to be used should be considered in the following


order: first line, vaginal; second line, abdominal; and third line, laparoscopic

True

False

Over the short- and long-term, there is higher patient satisfaction with
hysterectomy than endometrial ablation

True

False

Key points
Utilise the NICE 2007 Heavy menstrual bleeding guideline with its three-step
approach.
Step one is medical treatment with both hormonal and non-hormonal treatment.
Step two is minimally invasive uterus conserving surgery e.g. hysteroscopic
fibroid resection, endometrial ablation, transcervical resection of endometrium and
laparoscopic myomectomy.
Additional treatments have been added to step 2 involving newly developed
intervention in keeping with the above. It includes MRI guided focused USS therapy,
radiofrequency ablation of fibroids, uterine fibroid embolisation and laparoscopic
uterine artery occlusion.
Step three involves major surgical procedures e.g. abdominal myomectomy and
hysterectomy.
1477757342 frmReflectiveNote

Treatment of HMB

47
Assessment history:
Score
Total attempts: 0
Average score: 0%

Choose the treatment technique associated with the adverse effects listed in the items
below. Each option can be used once, more than once or not at all.

A: Tranexamic acid
B: Uterine artery embolisation (UAE)
C: Myomectomy
D: Non-steroidal anti-inflammatory drugs
E: Combined oral contraceptives
F: Hysterectomy
G: GnRHa (gonadotrophin-releasing hormone analogue)
H: Injected long-acting progestogens
I: Oral progestogen: norethisterone (15 mg) daily
J: Endometrial ablation
K: Oophorectomy at time of hysterectomy
L: Mirena (levonorgestrel-releasing intrauterine system)

Licensed for 5-year therapeutic usage. COMMON: irregular bleeding that may
last for over 6 months, hormone-related problems such as breast
tenderness, acne or headaches (generally minor and transient). LESS
COMMON: amenorrhoea. RARE: uterine perforation at the time of device
insertion.

Please Select

Only taken at the time of menses, no hormonal side effects, non-


contraceptive. LESS COMMON: indigestion, diarrhoea, headaches.

Please Select

Only taken at the time of menses, no hormonal side effects, non-


contraceptive. COMMON: indigestion; diarrhoea. RARE: worsening of asthma
in sensitive individuals, peptic ulcers with possible bleeding and peritonitis.

Please Select

Taken daily from days 1–21. Numerous contraceptive and non-contraceptive


beneficial effects (e.g. reduced risk of ovarian cancer and treatment for

48
estrogen-sensitive disorders, such as endometriosis and fibroids). COMMON:
hormonal side effects, mood changes, headaches, nausea, fluid retention,
breast tenderness. VERY RARE: deep vein thrombosis, stroke, heart attacks.

Please Select

Taken daily from days 5–26. COMMON: hormonal side effects, weight gain,
bloating, breast tenderness, headaches, acne (but all are usually minor and
transient) RARE: depression.

Please Select

Preparations are licensed for 3 months or 3 years. COMMON: weight gain,


irregular bleeding, amenorrhoea, premenstrual-like syndrome (including
bloating, fluid retention, breast tenderness). LESS COMMON: small loss of
bone mineral density, largely recovered when treatment discontinued.

Please Select

Preparations are licensed for 4 weeks or 3 months. COMMON: menopausal-


like symptoms (such as hot flushes, increased sweating, vaginal dryness)
which may require add-back therapy. LESS COMMON: osteoporosis,
particularly trabecular bone with longer than 6 months' use.

Please Select

Day-case surgical procedure, able to be performed under local anaesthetic in


outpatient setting. COMMON: vaginal discharge; increased period pain or
cramping (even if no further bleeding); need for additional surgery. LESS
COMMON: infection. RARE: uterine perforation (but very rare with second
generation techniques).

Please Select

Surgical procedure. LESS COMMON: adhesions (which may lead to pain


and/or impaired fertility), need for additional surgery, recurrence of uterine
fibroids, perforation (if performed by hysteroscopic route), infection. RARE:
haemorrhage.

Please Select

In patient surgical procedure. COMMON: infection. LESS COMMON:


intraoperative haemorrhage, damage to other abdominal organs, such as the

49
urinary tract or bowel, urinary dysfunction (frequent passing of urine and
incontinence). RARE: thrombosis (DVT and clot on the lung). VERY RARE:
death. (Complications are more likely if performed in the presence of
fibroids).

Please Select

In-patient surgical procedure. COMMON: menopausal-like symptoms.


Although beneficial outcome is substantial reduction in the risk of
developing ovarian cancer.

Please Select

Day-case procedure. COMMON: persistent vaginal discharge, post-


embolisation syndrome [pain, nausea, vomiting and fever (not involving
hospitalisation)]. LESS COMMON: need for additional surgery, premature
ovarian failure particularly in women over the age of 45 years, haematoma.
RARE: haemorrhage, non-target embolisation causing tissue necrosis,
infection causing septicaemia.

Please Select

AUB investigation and management


Assessment history:
Score
Total attempts: 0
Average score: 0%

The following items are clinical scenarios of women presenting with abnormal uterine
bleeding. For each item choose the best diagnostic procedure from the options listed
above that enables optimal management. Each option can be used once, more than
once or not at all.

A: Suspicious pathology. Physical examination and pelvic ultrasound required.


Recommend diagnostic hysteroscopy and endometrial biopsy and laparoscopy.
B: Suspicious pathology. Physical examination and pelvic ultrasound required.
Recommend diagnostic hysteroscopy and endometrial biopsy.
C: Suspicious pathology. Physical examination required. Obtain cervical smear. Prompt
referral to secondary care for urgent transvaginal pelvic ultrasound.
D: Suspicious pathology. Physical examination required. Obtain cervical smear.
Request full blood count.
E: No suspicious pathology. Physical examination not required. Commence first-line
treatment.

50
A 47-year-old woman presents with a 6 month history of intermenstrual
bleeding. Despite this, she still exhibits regular menstrual cycles without
associated pelvic pain and without any postcoital bleeding. She currently
uses condoms for contraception, is not clinically anaemic and all previous
cervical smears appeared normal. Her last smear was 1 year prior to
symptoms.

Please Select

A 61-year-old woman is concerned because over the last week she has been
experiencing vaginal blood loss described as a period-like. Her final
menstrual period was when she was around 50 and she has never taken HRT.

Please Select

A 41-year-old woman has a 2 year history of heavy menstrual bleeding. She


exhibits regular menstrual cycles without any associated pelvic pain and
does not suffer from intermenstrual or postcoital bleeding. She is currently
using condoms for contraception and is not clinically anaemic. All previous
cervical smears normal and her last smear was 1 year prior.

Please Select

After a 2 month episode of postmenopausal bleeding, a woman presents


with a cervical smear reporting 'atypical glandular cells of undetermined
significance, possibly endometrial cell origin'.

Please Select

Polyps arising from the uterine endometrium

Endometrial polyps are localised hyperplastic overgrowths of endometrial glands and


stroma which form projections over the endometrial surface.

Aetiology remains obscure, however it is proposed that, polyps lose their apoptotic cell
regulation and overexpress estrogen and progesterone receptors causing them to grow.
Endometrial polyps are rare before the age of 20 with the incidence steadily rising after
that, peaking in the 5th decade of life and then declining at menopause. Incidence
estimates are variable based on the population studied, however, 10% of asymptomatic
women (incidental on pelvic ultrasonography) and 24-41% of women with abnormal
uterine bleeding (AUB) have endometrial polyps.

Clinical presentation

 Asymptomatic
 Heavy menstrual bleeding

51
 Postmenopausal bleeding
 Prolapse through cervical ostium
 Abnormal vaginal discharge
 Breakthrough bleeding
 Infertility:
o large or multiple endometrial polyps can contribute to miscarriage and
infertility. Evidence also suggests that a polypectomy may improve spontaneous
conception rates in women with an endometrial polyp as the only factor contributing to
subfertility by normalising endometrial implantation factors
 Malignancy
o polyp size of >1 cm, abnormal uterine bleeding and postmenopausal
status are all independent risk factors for malignant polyps. Hysteroscopic markers for
malignant endometrial polyps include surface irregularities such as necrosis, vascular
irregularities and whitish thickened areas, which are indications for obtaining a
histological diagnosis.

Diagnostic modalities

 Pelvic ultrasound
 Saline infusion sonogram (SIS) – safe, well tolerated, rapid, minimally invasive,
highly sensitive and superior to ultrasound in diagnosing polyps
 Hysteroscopy – gold standard. Can be used to see and treat at the same sitting.

Enlarge

52
Enlarge

Management

 Hysteroscopic Polypectomy:
o symptomatic polyps
o asymptomatic polyps in postmenopausal women irrespective of size
o asymptomatic polyps in premenopausal women >1 cm in size.
 Observational treatment:
o asymptomatic polyps in premenopausal women ≤1 cm in size.
Annan JJ, Aquilina J, Ball E. The management of endometrial polyps in the 21st Century.
The Obstetrician & Gynaecologist 2012;14:33–38.

Key points
Endometrial polyps are localised hyperplastic overgrowths of endometrial glands
and stroma.
They can cause HMB, PMB, abnormal vaginal discharge and breakthrough
bleeding. When large or multiple they are implicated in subfertility.
Polyps of >1 cm, AUB and PBM are all risk factors for malignant polyps.
Diagnosis is achieved with USS, SIS and hysteroscopy.
They can be observed or removed hysteroscopically (symptomatic,
asymptomatic in postmenopausal patient, >1cm in size in an asymptomatic
premenopausal patient).

Adenomyosis

Background

53
Adenomyosis is a benign, common gynaecological condition causing heavy, painful
periods in premenopausal women who tend to be multiparous and between 40 and 50
years of age. Overall it is considered to contribute to approximately 10% of all cases of
HMB and 30% of all cases of HMB with dysmenorrhoea.

It is defined histologically (usually on a hysterectomy specimen) as the presence of


non-neoplastic endometrial glands and stroma in the myometrium. It is often
associated with hypertrophy and hyperplasia of the myometrium surrounding the
ectopic endometrial tissue. However, the clinical diagnosis of adenomyosis prior to
hysterectomy is often inaccurate. This is due to:

 variability and non-specific nature of symptoms, which tends to mimic any


combination of uterine fibroids, endometriosis and HMB due to ovulatory or endometrial
dysregulation
 variability of the sensitivity and specificity of pelvic ultrasound to diagnose
adenomyosis.

Screening investigations

Transvaginal ultrasound is useful as a screening tool in most patients. Studies on the


accuracy of TVS reported variable accuracy indices, with sensitivity and specificity
varying between 53%–89% and 50%–99% respectively, provided robust diagnostic
criteria are used (e.g. enlarged globular, regular uterus with no fibroids, myometrial
cystic areas and a decreased myometrial echogenicity) and an experienced
sonographer is undertaking the scan.

However, pelvic ultrasound becomes less predictive for adenomyosis in women with co-
existing fibroids. MRI has a higher diagnostic capability, irrespective of the presence or
absence of uterine fibroids, with estimates of 85% sensitivity/specificity in diagnosing
adenomyosis. The high cost and limited availability, however, hinder its routine use.

Diagnostic hysteroscopy is not considered an accurate method to diagnose


adenomyosis. However, it may provide some evidence of potential adenomyosis
through identification of the following:

 irregular endometrial surface


 pitting endometrial defects
 altered endometrial vascularisation
 cystic endometrial surface hemorrhagic lesions.

The role of invasive hysteroscopic or laparoscopic biopsy remains limited, with only
small series reported. The small number and size of biopsies obtained may be
insufficient to rule out the disease, especially given that the diagnosis may be
influenced by the numbers of uterine sections examined.

Clinical treatment

Management of adenomyosis uteri is hindered by the lack of a reliable, noninvasive


diagnostic test. No serum markers are currently available.

54
There are few randomised controlled trials assessing the effectiveness of treatment in
women with adenomyosis given the difficulties in accurate clinical diagnosis.
Nonetheless, there are varying levels of evidence to support the following treatments.

 Medical:
o nonhormonal therapy, including mefenamic and tranexamic acid, may be
effective for the symptomatic relief of menorrhagia
o low-dose, continuous combined oral contraceptives with withdrawal bleeds
every 4-6 months may be effective in relieving menorrhagia and dysmenorrhea
o high-dose continuous daily oral progestogens
o GnRHa agonist
o Mirena LNG-IUS.
 Uterus conserving:
o balloon endometrial ablation
o uterus-conserving minimally invasive treatments: uterine artery
embolisation, MRgFUS.
 Adenomyoma excision at the time of abdominal myomectomy.
 Laparoscopic myometrial electrocoagulation induces localised coagulation and
necrosis of adenomyosis uteri. This technique may be used in women > 40 years of age
who have completed their families, but who wish to avoid hysterectomy. Risks include
adhesion formation, bleeding resulting in a hysterectomy and difficulty in precise
application resulting in the weakening of the remainder of the myometrial tissue.
 Hysterectomy.

New treatments for adenomyosis being trialled include: aromatase inhibitors, selective
progesterone receptor modulators (e.g. asoprisnil) and bipolar radiofrequency ablation
via hysteroscopy or laparoscopy.

Imaging

Pelvic ultrasound imaging

55
Enlarge

MRI imaging

Key points
A benign condition causing heavy painful periods, contributing to 10% of HMB
and 30% of HMB with dysmenorrhoea.
Histologically it is defined as the presence of non-neoplastic endometrial glands
and stroma in the myometrium.
On USS it is represented by an enlarged globular regular uterus with no fibroids,
myometrial cystic areas and decreased myometrial echogenicity.
MRI diagnostic rates are higher than USS.
There are no serum markers available.
Management options include:
medical – mefanamic, tranexamic acid, low dose COC, high dose
continuous progesterones, GnRHa agonists and the Mirena LNG-IUS
uterus conserving – balloon ablation and uterine artery embolisation
adenomyoma excision at abdominal myomectomy
laparoscopic myometrial electrocoagulation
hysterectomy.

Leiomyoma (fibroids)

56
Uterine fibroids

Uterine fibroids are smooth muscle tumours of the uterus. While they are generally
benign, occasionally (<1%) malignant transformations can occur (leiomyosarcoma).

Uterine fibroids are generally age-related and are a commonly occurring pathology.
They are more common in African-Caribbean women than any other ethnicity. They
vary tremendously in size from millimetres to tens of centimetres and are associated
with heavy periods, pressure symptoms and occasionally pain.

Fibroids are responsive to the female hormones (estrogen and progesterone), generally
shrinking to a degree at menopause. They are classified as subserosal, intramural and
submucous according to their uterine location (see image below).

Site, size and number of fibroids are linked to the level of MBL.

57
Enlarge

Presentation, complications and investigations

Presentation

There is a wide spectrum on how women are diagnosed with fibroids. Most women are
referred due to symptoms. However, a significant number of women are incidentally
diagnosed with uterine fibroids and are essentially asymptomatic.

In general, fibroid treatment is initiated in women who are symptomatic of fibroid-


related symptoms, rather than women with asymptomatic fibroids. Fibroid symptoms
include the following:

 gynaecological - AUB, HMB, pelvic pain, dyspareunia, pelvic/abdominal mass


 anaemia due to HMB
 obstetric - infertility, miscarriage, abdominal pain (red degeneration of fibroids
midtrimester), preterm labour, malpresentation, caesarean delivery, postpartum
haemorrhage
 compression of organ systems - abdominal pressure-like effects on
gastrointestinal and urological tract and nerve entrapment like symptoms.

Complications

58
 Hyaline degeneration is relatively common and presents as painful enlarged
fibroids due to hyaline/cystic degeneration pathological process.
 Red degeneration (necrobiosis) occurs typically during pregnancy due to
infarction at mid-pregnancy.
 Calcification (‘womb stone’) - usually in postmenopausal women.
 Sarcomatous (malignant) change. Generally presents as a 0.2% risk. There is a
greater risk in women with multiple or rapidly growing fibroids, at advanced age, and if
there is a histology is leiomyosarcoma
 Infection (abscess) - relatively rare.
 Torsion of pedunculated fibroids.

Investigations

Assessment should follow the generic assessment process outlined earlier.

A large fibroid uterus can often be palpated as a firm pelvic mass. The ideal first-line
investigation is pelvic ultrasound (transvaginal and transabdominal), although MRI is
useful when planning surgery or as a baseline prior to uterine artery embolisation
(UAE).

Treatment hierarchy for women symptomatic of uterine fibroids

Treatment needs to be highly individualised for women with symptomatic uterine


fibroids. Key determinants include the following factors:

 uterine size and fibroid size/location/mapping number. Most first-step and


second-step treatments are unlikely to be successful in women with grossly
pathological (>20w size, multiple fibroids >5cm in size) multi-fibroid uteri
 desire for: fertility, uterus preservation and achieve definitive treatment and
awareness of risk of symptom recurrence/re-treatment if opting for uterus-conserving
treatment
 general medical health, age, BMI, previous surgery, previous fibroid treatments,
risk/benefit of treatment
 preference for focal fibroid treatment (e.g. myomectomy) or global uterine
treatment (e.g. UAE).

59
The NICE guideline for HMB (2007) has indicated that when surgery is necessary for
fibroid-related HMB, then the clinical case notes must demonstrate that there has been
due discussion and documentation of all treatment alternatives.

Issues like risk of re-treatment are significant for all uterus-conserving treatments;
around 20%–30% of all second-step treatments and abdominal myomectomy require
re-treatment in 2 years due to fibroid regrowth.

Treatment hierarchy Seeking contraception Wishing to


conceive
First step COCs Tranexamic acid
Oral progestogens NSAIDs
Injected progestogens
Mirena LNG-IUS
Short (<6 month) course of
GnRHa
Second step Hysteroscopic myomectomy Hysteroscopic
myomectomy
+/- Endometrial Laparoscopic
resection/ablation myomectomy
+/- Mirena LNG-IUS
Second step (minimally Uterine artery embolisation
invasive uterus-conserving Magnetic resonance-guided focused
treatments) ultrasonography
Laparoscopic uterine artery occlusion with/without
utero-ovarian occlusion

Transvaginal Doppler guided uterine artery


occlusion
Bipolar radiofrequency ablation (intrauterine
ultrasound-guided or laparoscopic-guided)
Third step Hysterectomy +/- bilateral Abdominal
salpingo-oophorectomy myomectomy
All focal fibroid treatments in RED font, Global uterine treatments in BLACK font

Preoperative GnRHa

There is evidence to support the use of a 3–4 month course of GnRHa prior to
myomectomy/hysterectomy as pre-treatment, as it:

 reduces fibroid size and uterine volume, thereby facilitating conversion of


midline vertical laparotomy skin incision to lower transverse abdominal incision in
women with large >24w sized multifibroid uteri
 improves preoperative haemoglobin levels
 reduces perioperative blood loss and transfusion requirements.

Hysterectomy versus abdominal myomectomy

Hysterectomy is a final step treatment to be considered if medical or uterus-conserving


options fail. However, hysterectomy can be considered as a first-step treatment if

60
symptoms are: severe (HMB, pressure-like effects, pain), there is significant pathology
(>20 week sized multifibroid uterus) that is unlikely to respond to other treatments;
there is a patient preference for definitive treatment and future fertility is not desired.

Another justification for hysterectomy is the combination of age and a significantly


pathological fibroid uterus. Abdominal myomectomy in a woman over 41 years with a
>20w sized multifibroid uterus is unlikely to substantially improve fertility (although it
would alleviate HMB and pain symptoms to some extent) irrespective of whether IVF is
undertaken.

Therefore, careful consideration should be given to hysterectomy as the preferred


treatment option in such circumstances as it carries lower surgical risks than abdominal
myomectomy and will achieve definitive symptom cure. For women below the age of
41, it would be justifiable to perform an abdominal myomectomy for either fertility
improvement (if that was desired) and/or alleviation of fibroid symptoms (HMB, pain),
or both, if the woman desired to preserve her uterus and rejected minimally invasive
uterus-conserving treatment alternatives.

Myomectomy and fertility

There is considerable debate on whether myomectomy improves fertility, although it is


established that fibroid presence (submucous and intramural) are associated with
adverse fertility and pregnancy outcomes. Based on the limited robust data available,
systematic reviews have concluded:

 removal of the intracavity component of the submucous fibroid improves fertility


(RR 1.72; 95% CI 1.13-2.58)
 subserosal fibroids do not affect fertility outcome
 intramural fibroids appear to decrease fertility, particularly those that distort the
uterine cavity. However, there is no definitive evidence that intramural myomectomy
improves fertility.

Minimally invasive uterus-conserving treatment

Uterine fibroid embolisation, MRI-guided focused ultrasound ablation and laparoscopic


uterine artery occlusion are grouped under a category called minimally invasive uterus
preserving treatments, which are considered alternatives to abdominal
myomectomy/hysterectomy.

These treatments are relatively new, but sufficient evidence on their safety and efficacy
has accumulated to warrant their recommendation within the NICE HMB guideline
(2007). Essentially these treatments share common characteristics of:

 effective in achieving fibroid symptom control (according to fibroid specific


quality of life tools) and patient satisfaction, but not necessarily achieve significant
reduction in fibroid or uterine size
 designed to be uterus-conserving therapies for women symptomatic of uterine
fibroids
 associated with shorter hospital stay, reduced recovery time, and less major
operative complications than abdominal myomectomy or hysterectomy

61
 uncertainty in their effects on fertility. Data conflicts on whether fertility is
reduced, improved or unchanged. Consequently, advice is that future fertility is not
advised after these treatments, although there are several hundred live births reported
following these treatments.

Transvaginal Doppler-guided uterine artery occlusion and bipolar radiofrequency


ablation treatments (intrauterine ultrasound-guided Vizablate®, laparoscopic-guided
Holt® system) are in the same minimally invasive uterus-conserving group of fibroid
treatments and are currently being evaluation.

National Institute for Health and Clinical Excellence. Heavy menstrual bleeding. NICE
clinical guideline 44. London: NICE; 2007.

Pretreatment
Ulipristal acetate (UPA) has been used predominantly in the context of managing HMB
in association with uterine fibroids. Ulipristal acetate is a selective progesterone
receptor modulator (SPRM), an orally active steroid compound, which reversibly blocks
the progesterone receptor in the endometrium and myometrium. The net effect is
inhibition of ovulation without significant effects on estradiol levels or glucocorticoid
activity.
UPA has been compared to treatment with a placebo (PEARL I), and leuprolide acetate
([3.75 mg 4 weekly] PEARL II) in the treatment of women with uterine fibroid associated
HMB. These studies have shown UPA to be effective in controlling uterine bleeding
related to myomas, reducing myoma size and having a good safety profile in the short
term. UPA was also found to be non-inferior to once monthly leuprolide acetate in
controlling uterine bleeding and was significantly less likely to cause hot flushes.
In clinical studies, SPRM administration has been associated with a pattern of benign,
nonphysiological, nonproliferative, histologic features of the endometrium termed P
receptor modulator associated endometrial changes (PAEC). This is entirely reversible
on stopping treatment.
In the UK, UPA is used in a dose of 5 mg once daily for up to 3 months for either
preoperative or intermittent treatment of moderate to severe symptoms of uterine
fibroids in adult women of reproductive age. Re-treatment should commence only when
menstruation has occurred. Repeated intermittent treatment has been studied up to 4
intermittent courses.
Donnez J, Tomaszewski J, Vázquez F, Bouchard P, Lemieszczuk B, Baró F, et al; PEARL II
Study Group. Ulipristal acetate versus leuprolide acetate for uterine fibroids. N Engl J
Med. 2012;366:421–32.
Donnez J, Tatarchuk TF, Bouchard P, Puscasiu L, Zakharenko NF, Ivanova T, et al;
PEARL I Study Group. Ulipristal acetate versus placebo for fibroid treatment before
surgery. N Engl J Med. 2012;366:409–20.
Donnez J, Donnez O, Matule D, Ahrendt HJ, Hudecek R, Zatik J, et al. Long-term medical
management of uterine fibroids with ulipristal acetate. Fertil Steril. 2016;105:165–173.

Hysteroscopic myomectomy
Submucous fibroid classification (ESGE)
Type Intramural extension
0 None

62
1 <50%
2 >50%
Wamsteker K, Emanuel MH, de Kruif JH. Transcervical hysteroscopic resection of
submucous fibroids for abnormal uterine bleeding: results regarding the degree of
intramural extension. Obstet Gynecol 1993; 82:736–40.
Fibroid size Intracavity fibroid 90–100%
HMB Decrease >80%
Fertility Increase 40–60%
Secondary treatment 2 year 10%; 5 year 30%
Complication rate 1–2%
Uterine perforation
Sepsis; intrauterine adhesions
Haemorrhage
Additional treatment TCRF may be combined with
Endometrial ablation +/- Mirena for improved effect on
HMB

Enlarge
Performing a hysteroscopic myomectomy using the hysteroscopic
resectoscope

63
Until recently, most hysteroscopic resectoscopes were MONOPOLAR (having a closed
cutting metal loop delivering electrocautery with patient return plate) and therefore
required non-ionising fluid (i.e. Glycine) as the intrauterine distension medium.
Newer hysteroscopic resectoscopes are BIPOLAR (having a special ‘incomplete’ loop
delivering electrocautery in the space around the loop, no patient return plate required)
and therefore may use normal saline as the intrauterine distension medium. An image
of a bipolar resectoscope equipment (Gynecare Alphascope) is depicted above.
The hysteroscopic loop, whether monopolar or bipolar, uses electrocautery to
simultaneously cut and coagulate the target tissue lesion; additional bursts of
coagulation energy can be targeted at endometrial surface bleeding points if bleeding
is occurring. The myoma is released by progressively shaving of the stalk (which
attaches the fibroid to the uterine wall and contains the fibroid’s blood supply) until it is
loosened.
During the procedure the loop of the resectoscope is placed at the most distant portion
and carefully withdrawn. Active resection should only occur when the resectoscope is
withdrawn from the cavity (in the direction of the surgeon). Futhermore, active
resection in the direction of the uterine fundus must be performed with extreme
caution as it increases the risk of both uterine perforation and damage to adjacent
organs.
A polyp or sponge holder forceps may be used to remove the mobilised free
intrauterine fibroid which should then sent for histological analysis. Further cervical
dilation is occasionally required to enable introduction of the forceps and final
withdrawal of the fibroid specimen.
Alternatively, once completely devascuarlised, the intrauterine fibroid may be left
within the uterus to undergo avascular degeneration. The necrosed fibroid will be
spontaneously expelled from the patient over a period of time as a blood stained
vaginal discharge. This alternative should only be considered if the surgeon is confident
that the fibroid polyp is benign and is considered appropriate only when there are
difficulties in dilating the cervix.
If bleeding occurs despite the electrocoagulation technique then a balloon catheter can
be inserted into the endometrial cavity postoperatively to tamponade the bleeding
sites. The balloon can then be deflated several hours after surgery and finally removed
when the bleeding has ceased.

Hyponatraemic fluid overload

Hyponatraemic fluid overload is a potentially life-threatening emergency, although


fortunately very rare (<1% of cases), presenting as confusion, collapse, seizure or
coma.
It occurs when intrauterine distending fluid (either gylcine if monopolar resectoscope or
saline if bipolar resectoscope) transmigrates through the uterus and enters the
circulatory system. Glycine is hypotonic relative to blood and therefore 1–1.5 L of
glycine entering the circulatory system can cause significant hyponatraemia.
However, normal saline is isotonic to blood, therefore up to 2 L of fluid can enter the
circulatory system without causing any significant dilutional hyponatraeimia.
Nonetheless, circulatory fluid overload can still occur if more than 2 L of fluid rapidly
enters the circulatory system, irrespective of whether it is glycine or saline source.
Hence, strict real-time fluid balance used during hysteroscopic myomectomy and the
procedure should be terminated if the estimated positive fluid deficit (fluid volume
instilled minus fluid volume summated from return effluent/drapes/floor/bucket)
exceeds 1 L for glycine or 2 L if normal saline is used as the uterine distension medium.
Abdominal myomectomy

64
The figure above shows a chronological sequence of photos taking from an abdominal
myomectomy procedure. Notice the use of a Foley catheter as a tourniquet around the
utero-upper cervix junction to create an avascular uterine body and therefore ‘dry
myomectomy’.

Abdominal myomectomy

 More than 80% improvement in AUB


 Uncertain if improves spontaneous fertility if over 41 years of age

65
 Very low conversion rate to hysterectomy (<1%)
 Comparable surgical morbidity risk to hysterectomy (organ damage, transfusion,
adhesion formation)
 Re-treatment rates after myomectomy: 20% over 2–5 years (symptom +/- fibroid
recurrence) depending on the woman’s age and how near she is to menopause.

Uterine artery embolisation / Uterine fibroid embolisation

Enlarge

Uterine artery embolisation (UAE) is an interventional radiological procedure where


both uterine arteries are blocked with particles injected via the femoral and uterine
arteries (see figure above). This causes the fibroids to shrink (necrosis due to vascular
occlusion) and is believed to have no permanent effect on the rest of the uterus.

Before performing the UAE treatment, ultrasound (ideally MRI) must be performed to
accurately measure and locate the fibroids, and to evaluate whether UAE is appropriate
for the woman. Interventional radiologists prefer to use MRI prior to UAE, to screen for
pedunculated/part pedunculated fibroid lesions (FIGO L0-L2, and, L6-L7), which carry
attendant risks of sloughing tumour into uterine/abdominal cavities and/or pelvic sepsis
following the procedure. Hysteroscopy and endometrial biopsy is also advised to rule
out submucosal fibroids and ensure benign endometrium.

Contraindications

 Active genitourinary infection


 Genital tract malignancy.

Relative contraindications

 Submucosal myomas (check hysteroscopy prior to UAE)

66
 Pedunculated myomas
 Recent GnRHa
 Previous UAE
 Postmenopausal status.

Procedure

A UAE is performed under conscious sedation with a local anesthetic at the site of
vascular catheterisation.

Effect on HMB

UAE alleviates symptoms of HMB in 79–93% of treated women. Additionally, women


treated with UAE have reported significant beneficial effects within 3 months of the
procedure.

UAE remains effective in reducing HMB and pressure-like symptoms for up to 5 years,
although recent studies have shown that there is a 20–29% chance of re-operation for
leiomyoma-related symptoms by this 5-year follow up.

Effect on fertility

The effect on fertility following UAE is unclear. There are no prospective studies
assessing fertility as a main outcome measure. Even so, there are at present around
100 cases of successful pregnancy (planned and unplanned) and live birth following
UAE.

Risks and/or adverse outcomes

 Severe pain
 Bleeding from puncture site
 Anaphylactic reactions to contrast dye
 Uterine endometritis, pelvic abscess, with or without infective sequelae
 Precipitate menopause (2–8% risk)
 Expulsion of necrotic fibroids or vaginal discharge (5–8% of cases) is more
common if fibroids are submucosal - this discharge may be unpleasant, prolonged and
distressing for the patient
 Unable to confidently exclude leiomyosarcoma presence, although follow-up
interval imaging may show irregular, persisting residual tissue mass even after a
technically successful procedure.

The future

Further studies (with longer term follow up) are needed to evaluate the effectiveness
and safety of UAE compared with myomectomy / hysterectomy (and other techniques
such as radiofrequency ablation).

X-ray fluoroscopy imaging

67
A) Immediately after UAE, B) Revascularisation 6 months after UAE

Fibroid shrinkage due to UAE on MRI

A) Pre-embolisation, B) 1 year post-embolisation (50% decrease in size)

Complications of UAE

68
Complications of UAE are based primarily on observational studies and data obtained
from various registries. These are divided into 3 major groups.

Immediate complications (<24 hours post procedure)

Local complications including pain, groin hematomas, contrast reactions and vasovagal
syncope have been reported. Non-target pelvic organ embolisation and pulmonary
embolism may occur.

Subacute complications (<30 days post procedure)

Post embolisation syndrome is a well-described phenomenon consisting of pain, fever,


nausea, leucocytosis and occasional malaise. It is a fairly common complication
associated with UAE and often needs to be distinguished from infections associated
with UAE. Treatment is supportive with analgesics, antipyretics, adequate hydration and
VTE prophylaxis. MRI may be needed in women with a protracted course of events to
rule out tissue necrosis and infection.

Infection (endometritis and sepsis) are rare complications associated with UAE. Uterine
necrosis is a rare complication, however, superimposed infection can be life
threatening. An MRI is prudent to help with the diagnosis. Most infections can be
managed with IV antibiotics and supportive treatment, although occasionally a
hysterectomy may be used in women with sepsis refractory to other measures.

Chronic complications (>30 days post procedure)

Fibroid expulsion is reported usually when UAE is used to treat submucous fibroids. A
partially attached and infarcted fibroid could require a further hysteroscopic resection.

Chronic vaginal discharge has been reported.

Non-target embolisation usually affects the ovaries and occasionally the vagina. When
the vaginal blood supply is compromised, the resultant vagina ischemia could result in
sexual dysfunction/dyspareunia. When the uteroovarian blood supply is affected, the
resultant ovarian ischemia is associated with follicular depletion and premature
menopause in older women (>45 years). In younger women, transient postmenopausal
symptoms have been reported. A high FSH level usually reverts to normal in 10–12
months. The potential for compromising fertility does exist and as such UAE is a
relatively contraindication in women wishing to retain their fertility.

Risk of miscarriage

UAE is associated with a higher cumulative risk of miscarriage; several fold higher than
the general population. Possible explanations include endometrial ischemia resulting in
an altered environment for implantation and fibroid migration and distortion of the
endometrial contour.

MRI-guided focused ultrasound ablation of uterine fibroids

69
Woman lying on ExAblate® 2000 (InSightec) focused ultrasound system ready to be
placed into an MRI unit
MRI-guided focused ultrasound ablation of
uterine fibroids
Fibroid size 20% at 2 years
AUB 60–70%
Higher NPV corresponds to greater fibroid size reduction and symptom relief at 12
months
Fertility Reported - mainly for single fibroid
ablations
Secondary treatment ? 30%
Complication rate 10% (pain, vaginal discharge)
1477769200 frmReflectiveNote

Key points
Fibroids are smooth muscle tumours of the uterus, usually benign but they can
have malignant transformations in >1% resulting in leiomyosarcoma.
They are classified by their location as subserosal, intramural and submucous.
Symptoms include AUB, HMB, dysparunia, anaemia and pressure symptoms. In
obstetrics they can cause subfertility, miscarriage, pain secondary to degeneration,
preterm labour, malpresentation and postpartum haemorrhage.
Treatment options vary depending on whether the woman wishes to conceive or
not.
Seeking contraception:
1st step – COC, oral/injected/IUS progestogens, short course of GnRHa
2nd step – hysteroscopic myomectomy +/- ablation +/- Mirena IUS.
Additional minimally invasive uterus-conserving treatment; UAE, MR focused
ultrasonography, laparoscopic uterine artery occlusion and bipolar radiofrequency
ablation
3rd step – hysterectomy +/- bilateral salpingo-ophrectomy.
Wishing to conceive:
1st step – tranexamic acid/NSAID’s
2nd step – hysteroscopic myomectomy, laparoscopic myomectomy.
Additional minimally invasive uterus-conserving treatments as above
3rd step – abdominal myomectomy.

Leiomyoma (fibroid)

70
Assessment history:
Score
Total attempts: 0
Average score: 0%

Clinical scenario

 A 38-year-old woman presents with a 2-year history of painful, heavy periods


 She bleeds for 10 days each month
 She feels her abdomen has become bloated over the same time period
 She feels tired most of the time, especially at the time of menses
 She has occasional constipation
 She is parity 2
 The GP has palpated a large, abdominal mass arising from the pelvis that feels
firm
 The GP has estimated the abodminal mass to be around 28 weeks of gestation
size and believes it to be consistent with a large fibroid uterus.

In the above clinical case, MRI scanning should be used as the first-line
diagnostic tool.

Medical treatments are likely to be successful if there is a large, multi-fibroid


uterus.

Hysterectomy may be used as a first-line treatment solely for HMB in the


presence of large fibroids and other fibroid related symptoms.

Answer whether the following statements are true or false.

UAE or myomectomy are appropriate treatment options if the woman wishes


to retain her fertility

True

False

GnRHa may be beneficially used prior to UAE, myomectomy or hysterectomy

True

False

Women who wish to become pregnant following myomectomy or UAE should


be cautioned about potential complications during pregnancy

71
True

False

Endometrial hyperplasia

Enlarge

Endometrial hyperplasia is defined as a proliferation of glands of irregular size and


shape with an increase in the glands/stroma ratio.

Endometrial hyperplasia is further classified into simple and complex hyperplasia based
on the complexity and crowding of the glandular framework:

 simple hyperplasia is a proliferative lesion with minimal glandular complexity


and crowding with abundant stroma between glands
 complex hyperplasia represents a proliferative lesion with severe glandular
complexity and crowding as well as minimal stroma between glands.

Endometrial hyperplasia can be further classified by the presence of cytological atypia


of the endometrial cells. Cytologic atypia refers to enlarged epithelial cells that are
hyperchromatic with prominent nucleoli and an increased nuclear-to-cytoplasmic ratio.
Cytologic atypia is the most important prognostic factor for progression to carcinoma.

There are four types of endometrial hyperplasia each varying in their predisposition to
malignant transformation.

Type of Cytological and Risk of Risk of spontaneous


endometrial architectural malignancy regression if untreated
hyperplasia criteria if untreated
Cystic and branching 1–3% 90% spontaneous
Simple non-atypical patterns progression regression rate

72
rate

Crowded or closely 1–5% 80–90% spontaneous


Complex non- opposed glands progression regression rate
atypical rate

Nuclear atypia and 10–25% 70% regression rate


Simple atypical hyperplasia progression
rate
Nuclear atypia and 30–50% 60% regression rate
Complex atypical hyperplasia progression
rate
 Less than 2% of hyperplasias without atypia progress to carcinoma and the mean
duration of progression to carcinoma takes almost 10 years.
 Atypical hyperplasia progresses to carcinoma in 23% of cases over a mean
duration of 4 years.
 Importantly, endometrial cancer may co-exist in 30–50% of atypical endometrial
hyperplasias.

Prevalence

Incidence of endometrial hyperplasia is age dependent. In women aged 40 years or


younger with AUB, only 1.3% were noted to have simple hyperplasia. The incidence
increases with age. In the postmenopausal age group with AUB, the incidence is 8–15%,
which includes cases of non-atypical and atypical hyperplasia.

Women with the following risk factors should have a comprehensive evaluation (pelvic
ultrasound and endometrial biopsy) if they present with AUB.

Risk factors for endometrial hyperplasia and malignant transformation of endometrial


hyperplasia include the following:

 perimenopausal and postmenopausal age groups


 obesity
 diabetes
 polycystic ovary syndrome (unopposed estrogens from anovulatory cycles)
 nulliparity
 hypertension
 unopposed estrogen therapy (risk is reduced but still remains increased if
progesterone opposition is added) in an intact uterus
 tamoxifen
 endogenous estrogens, e.g ovarian granulosa cell neoplasm
 family history of breast or endometrium cancer.

Atypical glandular cells of endometrial cell origin on a cervical smear indicate urgent
referral for hysteroscopic evaluation, as uterine cancer, endometrial hyperplasia or
cervical adenocarcinoma is present in 30–40% of such cases.

73
Treatment options

Remove any obvious exogenous estrogen sources. If taking oral or patch-based HRT,
then consideration could be given to Mirena LNG-IUS to both treat the endometrial
hyperplasia and continue to protect the uterus so that the patient can receive estrogen
only replacement therapy.

A similar approach could be used for tamoxifen-users, although Mirena-LNG IUS is only
licensed to be used as endometrial protection in HRT users and is not licensed for use in
tamoxifen users or women with endometrial hyperplasia.

Progestagens: oral progestagens (e.g. norethisterone) or Mirena LNG-IUS. Observational


studies have shown 90–95% regression in non-atypical hyperplasias.

Atypical hyperplasia should be treated by hysterectomy, and bilateral


salpingoophorectomy if in the perimenopausal or postmenopausal age group. High-
dose oral progestagens can be used for women deemed unfit for surgery.

Hysterectomy may be offered to non-atypical hyperplasia if they possess high risk


factors for malignant transformation (e.g. the patient is prescribed tamoxifen or has a
family history of endometrial cancer) or if they prefer this option to medical treatment
and interval endometrial surveillance.

In all cases of non-hysterectomy endometrial hyperplasia treatment, serial endometrial


biopsy surveillance should be instigated every 6 months (possibly every 3 months if
non-regressed hyperplasias). If there is non-regression or a persistence of non-atypical
hyperplasia beyond a defined interval (e.g. 6 or 12 months) or malignant
transformation (e.g. becomes atypical or endometrial cancer identified) then
hysterectomy and bilateral salpingoophorectomy should be performed.</

Tamoxifen and endometrial pathology

Women with breast cancer have an inherently increased risk of developing uterine
hyperplasia and uterine cancer irrespective of whether they are taking tamoxifen.

Around 10–40% of women taking tamoxifen incur uterine abnormalities ranging from
unscheduled AUB, polyps, atypical hyperplasia and endometrial cancer. Pelvic scans
may identify the classical subendometrial cystic pathology and thickened endometrium
associated with tamoxifen users, which can be misinterpreted as invasive endometrial
cancer.

A randomised controlled trial showed that tamoxifen induces a two-fold increased risk
of endometrial cancer. The absolute risk for endometrial cancer in tamoxifen-treated
women was 13.0 per 1000 compared to 5.4 per 1000 in women taking placebo.

There appears to be no benefit in routinely performing pelvic ultrasound and


endometrial biopsy in asymptomatic tamoxifen users in order to detect underlying
endometrial hyperplasia or cancer. However, tamoxifen users with AUB should all
undergo comprehensive evaluation, including at least a pelvic ultrasound and
endometrial biopsy and preferably hysteroscopy.

74
Key facts
AUB is the most common presenting symptom of endometrial hyperplasia.
Endometrial hyperplasia, although a benign condition, may be considered as a
precursor to endometrial cancer.
It is most often diagnosed in postmenopausal women, but women at any age
with unopposed estrogen from any source are at an increased risk for developing
endometrial hyperplasia, e.g. HRT or tamoxifen.
In the absence of cytological atypia, less than 2% of cases progress to cancer.
In the presence of cytological atypia, around 30–50% progress to endometrial
cancer or have co-existing endometrial cancer. Therefore, hysterectomy is
recommended for women with atypical endometrial hyperplasia.
Oral progestagens or Mirena LNG-IUS (unlicensed) have been used successfully
in the management of non-atypical endometrial hyperplasia.
The choice of treatment for endometrial hyperplasia is dependent on patient
age, the presence of cytologic atypia, the desire for future childbearing, and surgical
risk.

Endometrial hyperplasia
Assessment history:
Score
Total attempts: 0
Average score: 0%

Case scenario

 A 48-year-old woman presents to secondary care with a 3-year history of


worsening HMB with irregularity of her menstrual cycle
 She is para 2
 Her BMI is 42
 She feels tired and lethargic
 She is unresponsive to a 3-month course of tranexamic acid treatment
 Her cervical smear 12 months ago was normal.

The woman's pelvic ultrasound shows a uterus of 10 cm cavity length and endometrial
thickness of 15 mm (in menstrual phase of cycle) with no uterine structural
abnormalities and normal ovaries.

The following course of actions would be appropriate in the initial secondary


care management of this case.

Answer whether the following statements are true or false.

Consider listing directly for hysterectomy

True

False

75
Insert Mirena LNG-IUS

True

False

List for general anaesthetic day-case hysteroscopy and endometrial ablation

True

False

Undertake outpatient hysteroscopy and endometrial biopsy and discuss


risk/benefits of insertion of Mirena LNG-IUS with the woman prior to
hysteroscopy

True

False

Request a full blood count, endocrine profile (FSH, LH, testosterone,


prolactin, day 21 progesterone, estradiol, TFTs) and fasting glucose/HbA1c

True

False

von Willebrands disease

von Willebrands disease is a common haematological cause for menorrhagia. Combined


oral contraceptives are an effective treatment for menorrhagia in these patients as they
increase fibrinogen, prothrombin, factor VII, facrtor VIII and/or vWF. One study showed
that oral contraceptive pills were as efficacious as intranasal DDAVP in controlling
menorrhagia.

The levonorgestrel intrauterine system is an effective way of managing menorrhagia.

Endometrial ablation has also proven to be effective according to some studies.


However 3/7 women in one study needed hysterectomy.

Desmopressin acetate and vWF replacement may be needed in addition to hormonal


treatment to control menorrhagia in patients with von Willebrands disease

76
These patients should not be denied hysterectomy if needed. Advice from a
haematologist is essential if any surgical procedure is considered. The timing and need
for infusion will be determined by factor vWF and factor VIII levels preoperatively and
postoperatively.

Key points
VWD is a common cause of menorrhagia.
Management should be with COC (to increase fibrinogen, prothrombin, factor
VII, factor VII and vWF), other management options include Mirena IUS or endometrial
ablation if the woman's family is complete.
Desmopressin acetate and vWF replacement may be required.

Future trends

Reducing the need for hysterectomy: dilemma for healthcare provider,


clinician and woman

Hysterectomy exposes women to a risk (albeit low) of perioperative complications and


is therefore not considered an initial option, but as a ‘third step’ option following
attempted treatment with either Mirena LNG-IUS or endometrial ablation (EA) or both.

Promoting LNG-IUS and endometrial ablation therapies will help reduce the need for
hysterectomy in women with HMB and represents cost-effective treatment on direct
head-to-head treatment analysis. Cost savings are further amplified if outpatient local
anaesthetic, rather than day-case general anaesthetic, endometrial ablations are
performed. Uterine fibroid embolisation is becoming established as a treatment for
multi-fibroid, uterus-related HMB and is being offered by more hospital centres.

However, this impetus has recently been challenged by a cost-effectiveness analysis of


meta-analysed individual patient data. Despite longer hospital stay and time to
resumption of normal activities, more women were satisfied after hysterectomy
than after EA. Mirena was cheaper and more effective than first-generation ablation
techniques, with rates of satisfaction that were similar to second-generation EA.

Although hysterectomy is more expensive, it produces more quality-adjusted life years


(QALYs) relative to other remaining strategies and is likely to be considered cost
effective. The incremental cost-effectiveness ratio for hysterectomy compared with
Mirena is £1440 (€1633, $2350) per additional QALY. The incremental cost-effectiveness
ratio for hysterectomy compared with second-generation ablation is £970 per additional
QALY.

UK’s Health Technology Assessment, that funded the research, recommended that
future research should focus on evaluation of the clinical effectiveness and cost
effectiveness of the best second-generation EA technique under local anaesthetic
versus Mirena and types of hysterectomy such as laparoscopic supracervical
hysterectomy versus conventional hysterectomy and second generation endometrial
ablation.

77
Roberts TE, Tsourapas A, Middleton LJ, Champaneria R, Daniels JP, Cooper KG, et al.
Hysterectomy, endometrial ablation, and levonorgestrel releasing intrauterine system
(Mirena) for treatment of heavy menstrual bleeding: cost effectiveness analysis. BMJ
2011;342:d2202.

Middleton LJ, Champaneria R, Daniels JP, Bhattacharya S, Cooper KG, Hilken NH, et al.
Hysterectomy, endometrial destruction, and levonorgestrel releasing intrauterine
system (Mirena) for heavy menstrual bleeding: systematic review and meta-analysis of
data from individual patients. BMJ 2010;341:c3929.

Developing new medical treatments

Fibroid size HMB Fertility


GnRH antagonist: Decrease 30–40% Decrease Contraceptive
Ganirelix
Decrease 40% Decrease (risk of Contraceptive
Progesterone Decrease uterine endometrial
receptor size 40% hyperplasia)
antagonist:
Mifepristone

SPRMs: Decrease Decrease Contraceptive


Asoprisnil
Proellex
SERM: Unknown Decrease Contraceptive
Ormeloxifene
Aromatase Decrease Decrease Contraceptive
Inhibitor:
Anastrozole
Oral progestogen: Unknown May decrease Contraceptive
Dionogest
Combined pill: Unknown Decrease Contraceptive
Dienogest combined
with estradiol
valerate (Qlaira)

GnRH: gonadotrophin-releasing hormone; SPRM: selective progesterone receptor


modulator; SERM: selective estrogen receptor modulator

An estradiol valerate/dienogest combined oral contraceptive pill (E2V/DNG; Qlaira®) is


the first marketed natural estrogen contraceptive pill. The progestogen, dienogest, has
a potent endometrium stabilising effect. Furthermore, this pill is administered in a
unique dynamic dosing regimen, such that estradiol is reduced and dienogest is
increased,through a 26/2 pill cycle (26 active pills and 2 placebo pills).

78
Women treated with E2V/DNG have significantly shorter and lighter withdrawal
bleeding than women treated with an ethinylestradiol/levonorgestrel (EE/LNG) pill.
Around 20% of women on E2V/DNG reported absent withdrawal bleeding. Furthermore,
E2V/DNG has been shown to reduce menstrual bleeding in women with HMB. E2V/DNG
is the only oral contraceptive with a licensed indication for the management of heavy
menstrual bleeding.

Developing new surgical treatments for fibroids

 Laparoscopic bilateral uterine artery occlusion, with or without blockage of the


anastomosis between the uterine and ovarian vessels, for the management of
symptomatic uterine fibroids.
 Transvaginal Doppler-guided vascular clamp forthe management of symptomatic
uterine fibroids.
 Intrauterine ultrasound-guided radiofrequency ablation for uterine fibroids
(Gynesonics VizAblate® system, a transcervical device that combines ultrasound image
guidance with radiofrequency ablation to treat fibroids).
 Laparoscopic radiofrequency ablation of symptomatic uterine fibroids (Halt
Fibroid System™, Halt Medical, Inc.).
 Laparoscopic robotic hysterectomy and robotic myomectomy.

Lost IUCD case study


A 28-year-old woman is referred to you by her GP for advice regarding a lost IUCD. She
had a multi-load copper IUCD fitted 3 years ago and the 6-week post-coil check-up
suggested that it had been fitted satisfactorily.
However, the woman wishes to have the coil removed and on examination, no IUCD
threads are visible at the cervix in the GP surgery.

How would you manage this woman?


Lost IUCD - answer

How would you manage this woman?

Answer: This should cover:

1. Why does she wish the IUCD to be removed?

 Discuss likely procedures/investigations, contraceptive requirements and


alternative contraceptive methods before and after initial clinical examination in GOPD.
 Check LMP and consider pregnancy test if pregnancy suspected.

2. Likely to be intrauterine IUCD based on history and normal 6w coil check.

3. If unable to remove in OPD, establish location of IUCD, attempt removal:

 consider examining in colposcopy suite for better patient positioning and lighting
 probe the lower cervical canal to see if threads are visible. If so, grasp the thread
with polyp forceps
 often the threads and/or inferior aspect of body of IUCD, are withdrawn into the
internal os/isthmic portion of uterus and may be retrieved using long fine artery forceps
or specific IUCD plastic retriever.

79
4. If the removal attempt is unsuccessful:

 organise TV pelvic ultrasound (to see if IUCD is in uterus)


 if TV of the pelvis shows empty uterus, organise an X-ray of the pelvis to see if
IUCD is extrauterine/intraperitoneal.

5. If the IUCD is within the uterine cavity, arrange hysteroscopic removal:

 outpatient hysteroscopy with/without local anaesthetic is often the best option.


Remove coil under direct hysteroscopic version or blindly after first locating IUCD. May
need to dilate the cervix to facilitate IUCD removal
 in some rare cases, GA hysteroscopy is required to remove IUCD deeply
embedded in myometrium
 enquire whether the woman wishes simultaneous hysteroscopic sterilisation prior
to procedure.

6. If within the peritoneal cavity, arrange laparoscopic removal:

 at laparoscopy, most intraperitoneal IUCDs are visualised and grasped and


removed
 rarely, the IUCD is not visualised and X-ray image intensifier may help to localise
the IUCD
 rarely, laparotomy is required if the IUCD is deeply embedded in abdominal
viscus (bowel, bladder)
 enquire whether the woman wishes simultaneous laparoscopic sterilisation prior
to procedure.

HMB case study


A 46-year-old woman has a 3-year history of worsening heavy menstrual bleeding
(HMB) with regular cycles and dysmenorrhoea. She is parity 3 and has been sterilised.
She has tried COC, tranexamic and mefenamic acid in the past without success. She is
not anaemic and her cervical smear is normal and pelvic ultrasound shows no uterine
abnormalities. Her endometrial biopsy is normal.

What alternative treatments may be offered?


HMB case study - answer

What alternative treatments may be offered?

Answer: This should include:

1. History and general principles:

 establish the severity of HMB and how it has impacted on the woman's quality of
life
 what are her treatment expectations, e.g. need for definitive treatment, wishes
to avoid any type of surgery, wishes to avoid surgery that has prolonged convalescence
because of work/home factors
 shared decision making during counselling
 aim to individualise information on risks and benefits of treatment
alternatives and document of this process

80
 provide relevant patient information leaflets during counselling process.

2. Adopt a hierarchal approach to treatment (expectant, medical, uterus-


conserving surgery, hysterectomy categories etc).

3. Expectant management:

 may be an option in some peri-menopausal women, particularly if menopause


(complete cure) is imminent. Enquire about menopausal symptoms.

4. Medical therapies, risks and benefits

 Norethisterone (15 mg) daily from days 5 to 26 OR injected long-acting


progestogens. Lowers therapeutic efficacy compared to other alternatives, hormonal
side effects, compliance issues with oral therapy.
 Mirena LNG-IUS. Highly effective, initial 3–4 months erratic bleeding, other non-
contraceptive benefits (reduction in dysmenorrhoea). LNG-IUS presents the opportunity
to use estrogen replacement therapy with endometrial protection

5. Uterus-conserving surgery: second generation endometrial ablation

6. Advantages (compared with hysterectomy):

 variety of techniques (TBEA, MEA, Novasure) with procedure times <15 mins
 most can be performed in an outpatient setting, with or without local
anaesthetic, same day discharge, or as day case GA procedures
 most women do not need endometrial thinning or menstrual phase timing
 highly effective and safe (safer than previous first generation techniques)
 short period of convalescence (1–2 weeks) compared with hysterectomy.

Disadvantages (compared with hysterectomy):

 may not alleviate dysmenorrhoea


 only 10% dissatisfied by one year, but 20–30% dissatisfied by five years (due to
recurrence of symptoms, majority of these then requiring hysterectomy).

7. Hysterectomy

 Advantages – the definitive cure of menorrhagia and dysmenorrhoea, which


offers the highest patient satisfaction.
 Disadvantages – risk of major surgical morbidity and mortality, and prolonged
convalescence.
 Issues relating to oophorectomy – reduced risk of ovarian cancer and HRT.
 Issues relating to mode of hysterectomy, which may reduce convalescence time
(e.g. vaginal and laparoscopic methods compared with abdominal route), although this
is counterbalanced with evidence-based hierarchy based on operative complications
(i.e. higher bladder and ureteric complications with laparoscopic techniques).
 Issues relating to type of hysterectomy – subtotal and total.

Fibroid uterus case study

81
A 42-year-old woman is referred to you with a 4-year history of regular heavy periods
and pelvic pain. She is nulliparous.
The woman had been diagnosed to have a multi-fibroid uterus by another hospital and
has been recommended to have a hysterectomy. Abdominal examination reveals a 20w
sized fibroid uterus. She has been told that she has five fibroids, individually ranging in
size from 5 cm to 8 cm, at mostly intramural locations.
She is keen to preserve her uterus and wishes future fertility.

Outline your further investigations and justify your treatment options


accordingly.
Fibroid uterus case study answer

Outline your further investigations and justify your treatment options


accordingly.

Answer: This should include:

1. Baseline investigations in GOPD:

 FBC, endometrial biopsy, pelvic ultrasound; exclude hyperplasia/malignancy


 MRI is useful to map the location of fibroids exactly
 outpatient hysteroscopy to map presence of submucous fibroids if present or
uterine cavity distorted
 history or previous gynaecological treatments, surgery and medical co-
morbidities that could impact on surgical risk/benefit decision-making.

2. Realistic counselling about fertility prognosis:

 less than 1% chance of fertility, even if all fibroids are removed and IVF is
undertaken
 is uterus preservation justified given such poor fertility prognosis and significant
uterine pathology and symptoms?

3. Realistic counselling about treatment option and individualised


risk/benefit:

 adopt a hierarchal approach to treatment (expectant, medical, uterus-conserving


surgery, hysterectomy categories, etc) bearing in mind the aim is to improve HMB and
pain symptoms, as well as improve fertility if possible.

4. Expectant management:

 acceptable if not anaemic, no pressure effects of fibroids, baseline investigations


normal, menopause may be imminent, consider yearly ultrasound surveillance if large
fibroid uterus.

5. Medical therapies–risks and benefits:

 low therapeutic efficacy if large multi-fibroid uterus


 consider: tranexamic acid, NSAIDS, oral progestogens
 unsuitable for Mirena-LNG-IUS as enlarged uterine cavity.

82
6. Uterus-conserving - hysteroscopic submucous myomectomy:

 suitable if submucous fibroids <5 cm are identified


 consider outpatient or GA day-case procedure, with or without preoperative
GnRHa uterine preparation
 high therapeutic success rate for treating HMB and will improve fertility
(although absolute fertility chances remain <1%)
 relatively safe procedure with quick convalescence.

7. Uterus conserving - UAE:

 check for suitability, i.e. no submucous fibroids or narrow stalk subserosal


fibroids
 improves symptoms, future fertility not advised, preserves uterus
 low morbidity and speedy convalescence
 risk of symptom recurrence and re-treatment
 unsuitable for magnetic resonance-guided focused ultrasonography, laparoscopic
uterine artery occlusion.

8. Abdominal myomectomy

 unsuitable for laparoscopic myomectomy


 may be inappropriate in this perimenopausal age group as multiple fibroids,
enlarged uterus and fertility is unlikely to be significantly enhanced
 may be considered on basis of improving HMB and pain symptoms alone, and
woman wishing to keep uterus and prepared to accept: increased surgical risks and
morbidity of open abdominal myomectomy compared to hysterectomy; risk of re-
treatment due to fibroid re-growth.

9. Hysterectomy with/without BSO:

 preferred treatment if woman accepts uterus preservation is unlikely to benefit


her fertility and increases her risk of symptom recurrence and re-treatment irrespective
of uterus-preserving treatment chosen
 preferred treatment if definitive cure for HMB and pain is requested
 associated with lower morbidity, surgical risks and quicker convalescence than
abdominal myomectomy
 subtotal or total hysterectomy options, with or without BSO.

Endometrial hyperplasia case study


A 52-year-old woman has an endometrial biopsy for AUB. The histology shows
endometrial hyperplasia.

Outline how you would manage the patient and justify the treatment options
you would offer based on this result.

Endometrial hyperplasia case study answer

Outline how you would manage the patient and justify the treatment options
you would offer based on this result.

83
Answer: This should include:

1. Assess severity of AUB/HMB and the need to treat the woman's symptoms as
well as the need to treat the diagnosis of hyperplasia. Determine impact on quality of
life.

Consider hysteroscopy (particularly if thickened endometrium) as this enables:

 the removal of any intrauterine polyps, potentially reducing AUB symptoms


 improved sensitivity and specificity of diagnosis for endometrial cancer (that
may otherwise be missed)
 simultaneous insertion of Mirena LNG-IUS as therapy if agreed beforehand.

Assess risk factors that increase the woman's risk of malignant


transformation of endometrial hyperplasia:

 determined by non-atypical or atypical, simple and complex histology


 exogenous estrogen (tamoxifen, HRT) and or endogenous estrogen sources (e.g.
ovarian neoplasm, thus needs pelvic ultrasound)
 consider serum E2 and hormonal profile if ovarian cystic pathology identified
 personal and family history of breast/ovarian/uterine malignancy
 medical disorders (diabetes, hypertension, elevated BMI).

Ensure shared decision making during counselling:

 elicit patient preferences, explaining risks and benefits of all treatment


alternatives.

2. Adopt a hierarchal approach to treatment:

 expectant, medical (progestagens, Mirena LNG-IUS) and hysterectomy


categories.

3. Expectant management:

 acceptable if simple non-atypical hyperplasia and no other malignant


transformation risk factors
 offer serial endometrial pipelle histological surveillance (every 6 months)
 suitable if assessed to have low risk of malignancy (<2%), patient not wishing to
commence HRT (which could prolong or progress hyperplasia) and patient declines
medical treatment
 however, symptoms of AUB may persist or worsen if hyperplasia does not
regress.

4. Medical therapy. Suitable for both simple and complex non-atypical hyperplasia,
and no other malignant transformation risk factors:

 both are long-term treatments (at least 2 years)


 Mirena LNG-IUS has superior efficacy compared to oral progestins: 90% have
endometrial regression by 6 months with Mirena LNG-IUS whereas in oral progestins
treated women, 80% have regression

84
 oral progestagens–hormonal side effects, compliance issues; lower efficacy
 medical therapy is likely to improve AUB symptoms but is not curative
 lowers risk of malignancy if commencing treatment compared to no treatment
 requirement for serial outpatient endometrial pipelle biopsies (every 6 months).

5. Hysterectomy and BSO may be recommended if:

 atypical hyperplasia: high risk of malignant transformation with endometrial


cancer co-existing or developing in 30% of cases
 failed oral progestin/Mirena LNG medical therapy (histological non-regression or
hyperplasia persistence or progression despite 6 months of therapy) in non-atypical
hyperplasia
 strong patient preference, or is perceived to be at higher risk of malignant
transformation even though she has non-atypical hyperplasia, or symptoms markedly
affecting quality of life and definitive treatment is requested.

Assessment

A 50-year-old woman had a Mirena IUS inserted 2 years ago for heavy menstrual
periods. She was initially amenorrhoeic but has now developed heavy menstrual
bleeding again. Endometrial biopsy shows complex endometrial hyperplasia with
atypia.
What is the best treatment option?
Oral contraceptive pills in addition to Mirena in situ

Reinsert a Mirena coil

Total abdominal hysterectomy and bilateral salpingo-oophrectomy

Total abdominal hysterectomy

Tranexamic acid in addition to Mirena coil

A 55-year-old woman has been referred to the postmenopausal bleeding clinic following
an ultrasound organised by her GP for abdominal bloating. This showed the presence of
cystic spaces in the endometrium and an endometrial thickness of 15 mm. She has
previously used tamoxifen for 5 years for breast cancer.
What is the best management option?
High dose oral progestogens

Do nothing as she did not have any bleeding

Hysteroscopy and endometrial biopsy

Mirena IUS insertion

85
MRI

You review a 48-year-old woman in the menstrual disorders clinic who complains of a 3-
year history of heavy menstrual bleeding. She is a mother of four children, all born by
normal vaginal deliveries. Her menstrual cycle is every 29 days and the bleeding lasts
for 6 days. However, recently it has become associated with clots.
Cervical smears are up-to-date and her BMI is 39. You perform a transvaginal scan
which reveals a bulky uterus of 8 mm endometrial thickness and three intramural
fibroids of 2, 4 and 5 cm size respectively. On vaginal examination you find stage I
cystocele, stage II rectocele and stage II uterine descent.
The current waiting list for benign gynaecological surgery in your hospital is 4 months.
What is the next most appropriate step in her management?
Add on to the waiting list for laparoscopic-assisted vaginal hysterectomy

Arrange for pre-operative assessment for total abdominal hysterectomy

Counsel regarding NovaSure endometrial ablation

Insert Mirena intrauterine system and follow up in 6 months

Perform endometrial sampling

You are performing an outpatient hysteroscopy on a 62-year old woman who presents
with postmenopausal bleeding. Her menstrual cycles ceased at the age of 50. She used
combined HRT for 2 years afterwards.
A transvaginal scan reveals an endometrial thickness of 8 mm with a hyper-echoic
intracavitary shadow suggestive of an endometrial polyp. The size of the polyp was 16
mm in diameter.
What finding on hysteroscopy would make you consider the polyp as a
malignant lesion?
Broad-based avascular lesion

Haemorrhagic lesion with pus-like discharge

Multiple projections with mucous-like content

Smooth surface pedunculated polyp

Vascular surface

A 46-year-old para 2 who has completed her family presents with a history of painful
heavy menstrual bleeding in association with infrequent cycles (every 2–3 months) for
1 year. Her BMI is 44. She is currently on iron supplements for anaemia and is
prescribed proton pump inhibitors for GORD. She is otherwise fit and well.
Abdominopelvic examination is unremarkable. Pelvic ultrasound shows an endometrial

86
thickness of 12 mm with a bulky uterus and normal ovaries with no pelvic pathology. A
pipelle biopsy suggests a proliferative endometrium.
What treatment is most suited to her?
Combined oral contraceptive pill

GnRH Analogues

Mefenamic acid

Mirena IUS

Tranexemic acid

A 38-year-old Para 1 has a symptomatic fibroid uterus (5 cm intramural fibroid found on


ultrasound scan). She has been commenced on ulipristal acetate 5 mg OD for heavy
menstrual bleeding associated with this fibroid. She is keen to avoid surgery at present.

What is the electronic Medicines Compendium (eMC) guidance for the


maximum permissible dose of ulipristal acetate in the UK for this purpose?
2.5 mg OD for 6 months for 4 treatment cycles

5 mg BD for a maximum of 3 months for 3 treatment cycles

5 mg OD for a maximum of 3 months for 4 treatment cycles

5 mg OD for a maximum of 6 months for 4 treatment cycles

10 mg OD for a maximum of 3 months for 4 treatment cycles

A woman who has recently had a uterine artery embolisation performed for a fibroid
uterus (18 weeks size – intramural and submucous fibroids) presents to the emergency
department with fever, nausea, vomiting, and foul smelling vaginal discharge.
Which investigation is best suited to guide further management?
CT scan of the abdomen and pelvis

Hysteroscopy

MRI of the abdomen and pelvis

Transabdominal ultrasound

Transvaginal ultrasound

87

Potrebbero piacerti anche